Hinkle 15th edition Test bank Ch. 34, Ch. 35, Ch. 36 Ch. 37, Ch. 38, CH. 39, Ch. 40, Ch. 41, Ch 42, Ch. 43, Ch. 44

Lakukan tugas rumah & ujian kamu dengan baik sekarang menggunakan Quizwiz!

39) A female client has presented to the emergency department with right upper quadrant pain; the health care provider has ordered abdominal ultrasound to rule out cholecystitis. The client expresses concern to the nurse about the safety of this diagnostic procedure. How should the nurse best respond? A. "Abdominal ultrasound is very safe, but it can't be performed if you're pregnant." B. "Abdominal ultrasound poses no known safety risks of any kind." C. "Current guidelines state that a person can have up to 3 ultrasounds per year." D. "Current guidelines state that a person can have up to 6 ultrasounds per year."

"Abdominal ultrasound poses no known safety risks of any kind."

A client with gastritis required hospital treatment for an exacerbation of symptoms and receives a subsequent diagnosis of pernicious anemia due to malabsorption. When planning the client's continuing care in the home setting, what assessment question is most relevant? A. "Does anyone in your family have experience at giving injections?" B. "Are you going to be anywhere with strong sunlight in the next few months?" C. "Are you aware of your blood type?" D. "Do any of your family members have training in first aid?"

"Does anyone in your family have experience at giving injections?"

A client presents at the ambulatory clinic reporting recurrent sharp stomach pain that is relieved by eating. The nurse suspects that the client may have an ulcer. How should the nurse explain the formation and role of acid in the stomach to the client? A. "Hydrochloric acid is secreted by glands in the stomach in response to the actual or anticipated presence of food." B. "As digestion occurs in the stomach, the stomach combines free hydrogen ions from the food to form acid." C. "The body requires an acidic environment in order to synthesize pancreatic digestive enzymes; the stomach provides this environment." D. "The acidic environment in the stomach exists to buffer the highly alkaline environment in the esophagus."

"Hydrochloric acid is secreted by glands in the stomach in response to the actual or anticipated presence of food."

The management of the client's gastrostomy is an assessment priority for the home care nurse. What statement would indicate that the client is managing the tube correctly? A. "I clean my stoma twice a day with alcohol." B. "The only time I flush my tube is when I'm putting in medications." C. "I flush my tube with water before and after each of my medications." D. "I try to stay still most of the time to avoid dislodging my tube."

"I flush my tube with water before and after each of my medications."

A nurse is working with a client with rheumatic disease who is being treated with salicylate therapy. What statement would indicate that the client is experiencing adverse effects of this drug?

"I have this ringing in my ears that just won't go away."

A nurse is caring for a client who has had a total hip replacement. The nurse is reviewing health education prior to discharge. Which of the client's statements would indicate to the nurse that the client requires further teaching?

"I will need my husband to assist me in getting off the low toilet seat at home."

A client with systemic lupus erythematosus (SLE) is preparing for discharge. The nurse knows that the client has understood health education when the client makes what statement?

"I'll make sure to monitor my body temperature on a regular basis."

A client seeking care because of recurrent heartburn and regurgitation is subsequently diagnosed with a hiatal hernia. Which of the following should the nurse include in health education? A. "Drinking beverages after your meal, rather than with your meal, may bring some relief." B. "It's best to avoid dry foods, such as rice and chicken, because they're harder to swallow." C. "Many clients obtain relief by taking over-the-counter antacids 30 minutes before eating." D. "Instead of eating three meals a day, try eating smaller amounts more often."

"Instead of eating three meals a day, try eating smaller amounts more often."

A client with a right tibial fracture is being discharged home after having a cast applied. What instruction should the nurse provide in relationship to the client's cast care?

"Keep your right leg elevated above heart level."

The nurse is providing a client with the supplies necessary to perform two hemoccult tests on the client's stool. What instruction should the nurse give this client? A. "If possible, fast for 12 hours before collecting a sample." B. "Take all your medications except the antihypertensive ones." C. "Don't eat highly acidic foods 72 hours before you start the test." D. "Mail the paper slides to the clinic once you've collected the samples."

"Mail the paper slides to the clinic once you've collected the samples."

A client was brought to the emergency department after a fall. The client is taken to the operating room to receive a right hip prosthesis. In the immediate postoperative period, what health education should the nurse emphasize?

"Make sure you don't bring your knees close together."

A client comes to the clinic reporting pain in the epigastric region. What statement by the client is specific to the presence of a duodenal ulcer? A. "My pain resolves when I have something to eat." B. "The pain begins right after I eat." C. "I know that my father and my grandfather both had ulcers." D. "I seem to have bowel movements more often than I usually do."

"My pain resolves when I have something to eat."

A client is undergoing diagnostic testing to determine the etiology of recent joint pain. The client asks the nurse about the difference between osteoarthritis (OA) and rheumatoid arthritis (RA). What is the best response by the nurse?

"OA is considered a noninflammatory joint disease. RA is characterized by inflamed, swollen joints."

The nurse is teaching the client on bed rest to perform quadriceps setting exercises. Which instruction should the nurse give the client?

"Push the knees into the mattress."

A client who suffered a stroke had an NG tube inserted to facilitate feeding shortly after admission. The client has since become comatose and the client's family asks the nurse why the health care provider is recommending the removal of the client's NG tube and the insertion of a gastrostomy tube. What is the nurse's best response? A. "It eliminates the risk for infection." B. "Feeds can be infused at a faster rate." C. "Regurgitation and aspiration are less likely." D. "It allows caregivers to provide personal hygiene more easily."

"Regurgitation and aspiration are less likely."

A client has sought care because of recent dark-colored stools. As a result, a fecal occult blood test has been ordered. The nurse should give what instructions to the client? A. "Take no NSAIDs within 72 hours of the test." B. "Take prescribed medications as usual." C. "Avoid over-the-counter (OTC) vitamin C supplements." D. "Do not use fiber supplements before the test."

"Take no NSAIDs within 72 hours of the test."

A client is admitted to the orthopedic unit in skeletal traction for a fractured proximal femur. Which explanation should the nurse give the client about skeletal traction?

"Traction involves passing a pin through the bone."

A nurse is providing preprocedure education for a client who will undergo a lower GI tract study the following week. What should the nurse teach the client about bowel preparation? A. "You'll need to fast for at least 18 hours prior to your test." B. "Starting today, take over-the-counter (OTC) stool softeners twice daily." C. "You'll need to have enemas the day before the test." D. "For 24 hours before the test, insert a glycerin suppository every 4 hours."

"You'll need to have enemas the day before the test."

A client has been brought to the emergency department with abdominal pain and is subsequently diagnosed with appendicitis. The client is scheduled for an appendectomy but questions the nurse about how a person's health is affected by the absence of the appendix. How should the nurse best respond? A. "Your appendix doesn't play a major role in health, so you won't notice any difference after your recovery from surgery." B. "The surgeon will encourage you to limit your fat intake for a few weeks after the surgery, but your body will then begin to compensate." C. "Your body will absorb slightly fewer nutrients from the food you eat, but you won't be aware of this." D. "Your small intestine will adapt over time to the absence of your appendix."

"Your appendix doesn't play a major role in health, so you won't notice any difference after your recovery from surgery."

The nurse is providing care for a client whose peptic ulcer disease will be treated with a Billroth I procedure (gastroduodenostomy). Which statement(s) by the client indicates effective knowledge of the procedure? Select all that apply. A. "I will be at risk of developing diarrhea, nausea, and feeling light-headed after eating." B. "It is likely that I will need to receive nutrition directly into my veins." C. "One of my nerves, the vagus nerve, may be cut during the surgery." D. "I can eat a normal diet again after 3 to 5 weeks." E. "This surgery will remove part of my stomach and colon."

- "I will be at risk of developing diarrhea, nausea, and feeling light-headed after eating." - "One of my nerves, the vagus nerve, may be cut during the surgery."

An adult client with a history of dyspepsia has been diagnosed with chronic gastritis. The nurse's health education should include what guidelines? Select all that apply. A. Avoid drinking alcohol B. Adopt a low-residue diet C. Avoid nonsteroidal anti-inflammatories D. Take calcium gluconate as prescribed E. Prepare for the possibility of surgery

- Avoid drinking alcohol - Avoid nonsteroidal anti-inflammatories

A nurse is teaching an educational class to a group of older adults at a community center. In an effort to prevent osteoporosis, the nurse should encourage participants to ensure that they consume the recommended intake of what nutrients? Select all that apply.

- Calcium - Vitamin D

A nurse is creating a care plan for a client who is receiving parenteral nutrition. The client's care plan should include nursing action(s) relevant to what potential complications? Select all that apply. A. Dumping syndrome B. Clotted or displaced catheter C. Pneumothorax D. Hyperglycemia E. Line sepsis

- Clotted or displaced catheter - Pneumothorax - Hyperglycemia - Line sepsis

A nurse is caring for a client who is being assessed following reports of severe and persistent low back pain. The client is scheduled for diagnostic testing in the morning. Which of the following are appropriate diagnostic tests for assessing low back pain? Select all that apply.

- Computed tomography (CT) - Magnetic resonance imaging (MRI) - Ultrasound - . X-ray

A client with pancreatic cancer has been scheduled for a pancreaticoduodenectomy (Whipple procedure). During health education, the client should be informed that this procedure will involve the removal of which of the following? Select all that apply. A. Gallbladder B. Part of the stomach C. Duodenum D. Part of the common bile duct E. Part of the rectum

- Gallbladder - Part of the stomach - Duodenum - Part of the common bile duct

. A nurse is providing care for a client who has a recent diagnosis of Paget disease. When planning this client's nursing care, what should interventions address? Select all that apply.

- Impaired physical mobility - Acute pain - Disturbed auditory sensory perception - Risk for injury

An adult client has been diagnosed with diverticular disease after ongoing challenges with constipation. The client will be treated on an outpatient basis. What components of treatment should the nurse anticipate? Select all that apply. A. Anticholinergic medications B. Increased fiber intake C. Enemas on alternating days D. Reduced fat intake E. Fluid reduction

- Increased fiber intake - Reduced fat intake

A nurse is caring for a client in the late stages of esophageal cancer. The nurse should plan to prevent or address what characteristic(s) of this stage of the disease? Select all that apply. A. Perforation into the mediastinum B. Development of an esophageal lesion C. Erosion into the great vessels D. Painful swallowing E. Obstruction of the esophagus

- Perforation into the mediastinum - Erosion into the great vessels - Obstruction of the esophagus

41) A nurse is preparing to discharge a client home on parenteral nutrition. What should an effective home care teaching program address? Select all that apply. A. Preparing the client to troubleshoot for problems B. Teaching the client and family strict aseptic technique C. Teaching the client and family how to set up the infusion D. Teaching the client to flush the line with sterile water E. Teaching the client when it is safe to leave the access site open to air

- Preparing the client to troubleshoot for problems - Teaching the client and family strict aseptic technique - Teaching the client and family how to set up the infusion

A client with end-stage liver disease has developed hypervolemia. What nursing interventions would be most appropriate when addressing the client's fluid volume excess? Select all that apply. A. Administering diuretics B. Administering calcium channel blockers C. Implementing fluid restrictions D. Implementing a 1500 kcal/day restriction E. Enhancing client positioning

- administering diuretics - implementing fluid restrictions - enhancing client positioning

A client's health care provider has ordered a "liver panel" in response to the client's development of jaundice. When reviewing the results of this laboratory testing, the nurse should expect to review what blood tests? Select all that apply. A. Alanine aminotransferase (ALT) B. C-reactive protein (CRP) C. Gamma-glutamyl transferase (GGT) D. Aspartate aminotransferase (AST) E. B-type natriuretic peptide (BNP)

- alanine aminotransferase (ALT) - gamma-glutamic transferase (CGT - aspartame aminotransferase (AST)

The nurse is caring for a client with a duodenal ulcer and is relating the client's symptoms to the physiologic functions of the small intestine. What do these functions include? Select all that apply. A. Secretion of hydrochloric acid (HCl) B. Reabsorption of water C. Secretion of mucus D. Absorption of nutrients E. Movement of nutrients into the bloodstream

-Secretion of mucus -Absorption of nutrients -Movement of nutrients into the bloodstream

A client has been experiencing significant psychosocial stress in recent weeks. The nurse is aware of the hormonal effects of stress, including norepinephrine release. Release of this substance would have what effect on the client's gastrointestinal function? Select all that apply. A. Decreased motility B. Increased sphincter tone C. Increased enzyme release D. Inhibition of secretions E. Increased peristalsis

-decreased motility -increased sphincter tone -Inhibit of secretion

A nurse is caring for a client admitted with a suspected malabsorption disorder. The nurse knows that one of the accessory organs of the digestive system is the pancreas. What digestive enzymes does the pancreas secrete? Select all that apply. A. Pepsin B. Lipase C. Amylase D. Trypsin E. Ptyalin

-lipase -amylase -trypsin

The nurse educator is reviewing the blood supply of the GI tract with a group of medical nurses. The nurse is explaining the fact that the veins that return blood from the digestive organs and the spleen form the portal venous system. What large veins will the nurse list when describing this system? Select all that apply. A. Splenic vein B. Inferior mesenteric vein C. Gastric vein D. Inferior vena cava E. Saphenous vein

-splenic vein -inferior mesenteric vein -gastric vein

CHAPTER 42 - Assessment of pt with obesity

.

CHAPTER 43 - Hepatic Disorders

.

CHAPTER 44 - Biliary disorders

.

Chapter 41 - mgmt of Intestinal and rectal disorder

.

CHAPTER 39

...

CHAPTER 40 - Gastric and duodenal disorder

...

A client returns to the unit after a neck dissection. The surgeon placed a Jackson-Pratt drain in the wound. When assessing the wound drainage over the first 24 postoperative hours the nurse would notify the health care provider immediately for what finding? A. Presence of small blood clots in the drainage B. 60 mL of milky or cloudy drainage C. Spots of drainage on the dressings surrounding the drain D. 120 mL of serosanguinous drainage

60 mL of milky or cloudy drainage

A nurse who provides care in an ambulatory clinic integrates basic cancer screening into admission assessments. What client most likely faces the highest immediate risk of oral cancer? A. A 65-year-old man with alcoholism who smokes B. A 45-year-old woman who has type 1 diabetes and who wears dentures C. A 32-year-old man who is obese and uses smokeless tobacco D. A 57-year-old man with GERD and dental caries

A 65-year-old man with alcoholism who smokes

A client has come to the clinic reporting pain just above her umbilicus. When assessing the client, the nurse notes Sister Mary Joseph nodules. The nurse should refer the client to the primary provider to be assessed for what health problem? A. A GI malignancy B. Dumping syndrome C. Peptic ulcer disease D. Esophageal/gastric obstruction

A GI malignancy

A client's assessment and diagnostic testing are suggestive of acute pancreatitis. When the nurse is performing the health interview, what assessment question(s) addresses likely etiologic factors? Select all that apply. A. "How many alcoholic drinks do you typically consume in a week?" B. "Have you ever been tested for diabetes?" C. "Have you ever been diagnosed with gallstones?" D. "Would you say that you eat a particularly high-fat diet?" E. "Does anyone in your family have cystic fibrosis?"

A How many alcoholic drinks do you typically consume in a week?" B "Have you ever been tested for diabetes?" C "Have you ever been diagnosed with gallstones?" D "Would you say that you eat a particularly high-fat diet?"

A nurse is caring for a patient whose cancer metastasis has resulted in bone pain. Which of the following are typical characteristics of bone pain?

A dull, deep ache that is boring in nature

A client has come to the clinic reporting blood in the stool. A fecal occult blood test is performed but is negative. Based on the client's history, the health care provider suggests a colonoscopy, but the client refuses, citing a strong aversion to the invasive nature of the test. What other test might the provider order to check for blood in the stool? A. A laparoscopic intestinal mucosa biopsy B. A fecal immunochemical test (FIT) C. Computed tomography (CT) D. Magnetic resonance imagery (MRI)

A fecal immunochemical test (FIT)

A nurse is conducting health screening with a diverse group of clients. Which client likely has the most risk factors for developing hemorrhoids? A. A 45-year-old teacher who stands for 6 hours per day B. A pregnant woman at 28 weeks' gestation C. A 37-year-old construction worker who does heavy lifting D. A 60-year-old professional who is under stress

A pregnant woman at 28 weeks' gestation

A nurse's assessment of a client's knee reveals edema, tenderness, muscle spasms, and ecchymosis. The client states that 2 days ago the client ran in a long-distance race and now it "really hurts to stand up." The nurse should plan care based on the belief that the client has experienced what injury?

A second-degree strain

The surgical nurse is admitting a client from postanesthetic recovery following the client's below-the-knee amputation. The nurse recognizes the client's high risk for postoperative hemorrhage and should keep what equipment at the bedside?

A tourniquet

A triage nurse in the emergency department is assessing a client who presented with reports of general malaise. Assessment reveals the presence of jaundice and increased abdominal girth. What assessment question best addresses the possible etiology of this client's presentation? A. "How many alcoholic drinks do you typically consume in a week?" B. "To the best of your knowledge, are your immunizations up to date?" C. "Have you ever worked in an occupation where you might have been exposed to toxins?" D. "Has anyone in your family ever experienced symptoms similar to yours?"

A) "How many alcoholic drinks do you typically consume in a week?"

A nurse is caring for a client with hepatic encephalopathy. While making the initial shift assessment, the nurse notes that the client has a flapping tremor of the hands. The nurse should document the presence of what sign of liver disease? A. Asterixis B. Constructional apraxia C. Fetor hepaticus D. Palmar erythema

A) Asterixis

A 55-year-old female client with hepatocellular carcinoma (HCC) is undergoing radiofrequency ablation. The nurse should recognize what goal of this treatment? A. Destruction of the client's liver tumor B. Restoration of portal vein patency C. Destruction of a liver abscess D. Reversal of metastasis

A) Destruction of the client's liver tumor

A group of nurses have attended an in-service on the prevention of occupationally acquired diseases that affect health care providers. What action has the greatest potential to reduce a nurse's risk of acquiring hepatitis C in the workplace? A. Disposing of sharps appropriately and not recapping needles B. Performing meticulous hand hygiene at the appropriate moments in care C. Adhering to the recommended schedule of immunizations D. Wearing an N95 mask when providing care for clients on airborne precautions

A) Disposing of sharps appropriately and not recapping needles

During a health education session, a participant has asked about the hepatitis E virus. What prevention measure should the nurse recommend for preventing infection with this virus? A. Following proper hand-washing techniques B. Avoiding chemicals that are toxic to the liver C. Wearing a condom during sexual contact D. Limiting alcohol intake

A) Following proper hand-washing techniques

A nurse is assisting with serving dinner trays on the unit. Upon receiving the dinner tray for a client admitted with acute gallbladder inflammation, the nurse will question which of the following foods on the tray? A. Fried chicken B. Mashed potatoes C. Dinner roll D. Tapioca pudding

A) Fried chicken

A client has been scheduled for an ultrasound of the gallbladder the following morning. What should the nurse do in preparation for this diagnostic study? A. Have the client refrain from food and fluids after midnight. B. Administer the contrast agent orally 10 to 12 hours before the study. C. Administer the radioactive agent intravenously the evening before the study. D. Encourage the intake of 64 ounces of water 8 hours before the study.

A) Have the client refrain from food and fluids after midnight.

A nurse educator is teaching a group of recent nursing graduates about their occupational risks for contracting hepatitis B. What preventative measures should the educator promote? Select all that apply. A. Immunization B. Use of standard precautions C. Consumption of a vitamin-rich diet D. Annual vitamin K injections E. Annual vitamin B12 injections

A) Immunization B) Use of standard precautions

A nurse on a solid organ transplant unit is planning the care of a client who will soon be admitted upon immediate recovery following liver transplantation. What aspect of nursing care is the nurse's priority? A. Implementation of infection-control measures B. Close monitoring of skin integrity and color C. Frequent assessment of the client's psychosocial status D. Administration of antiretroviral medications

A) Implementation of infection-control measures

A client's abdominal ultrasound indicates cholelithiasis. When the nurse is reviewing the client's laboratory studies, what finding is most closely associated with this diagnosis? A. Increased bilirubin B. Decreased serum cholesterol C. Increased blood urea nitrogen (BUN) D. Decreased serum alkaline phosphatase level

A) Increased bilirubin

38) The nurse's review of a client's most recent laboratory results indicates a bilirubin level of 3.0 mg/dL (51 mmol/L). The nurse assesses the client for: A. jaundice. B. bleeding. C. malnutrition. D. hypokalemia.

A) Jaundice

A client is being discharged after a liver transplant and the nurse is performing discharge education. When planning this client's continuing care, the nurse should prioritize what risk diagnosis? A. Risk for infection related to immunosuppressant use B. Risk for injury related to decreased hemostasis C. Risk for unstable blood glucose related to impaired gluconeogenesis D. Risk for contamination related to accumulation of ammonia

A) Risk for infection related to immunosuppressant use

A 37-year-old client presents at the emergency department (ED) reporting nausea and vomiting and severe abdominal pain. The client's abdomen is rigid, and there is bruising to the client's flank. The client's spouse states that the client was on a drinking binge for the past 2 days. The ED nurse should assist in assessing the client for what health problem? A. Severe pancreatitis with possible peritonitis B. Acute cholecystitis C. Chronic pancreatitis D. Acute appendicitis with possible perforation

A) Severe pancreatitis with possible peritonitis

A client has developed hepatic encephalopathy secondary to cirrhosis and is receiving care on the medical unit. The client's current medication regimen includes lactulose four times daily. What desired outcome should the nurse relate to this pharmacologic intervention? A. Two to three soft bowel movements daily B. Significant increase in appetite and food intake C. Absence of nausea and vomiting D. Absence of blood or mucus in stool

A) Two to three soft bowel movements daily

A client with long-standing obesity has been prescribed phentermine/topiramate-ER. What statement by the client suggests that further health education is necessary? A. "I'm so relieved to start this medication. I really don't like having to exercise or change what I eat." B. "It's hard to believe that there are actually medications that can treat obesity." C. "I'm a bit nervous to start this medication because I know I'll need blood tests sometimes." D. "I'm going to have to do some rearranging of my finances to make sure I can afford this medication."

A. "I'm so relieved to start this medication. I really don't like having to exercise or change what I eat."

The nurse has completed the admission assessment of a client and has determined that the client's body mass index (BMI) is 33.5 kg/m2 . What health promotion advice should the nurse provide to the client? A. "It would be very helpful if you could integrate more physical activity into your routine." B. "You're considered to be overweight, so you should be diligent about maintaining a healthy diet." C. "You might want to consider some of the surgical options that have been developed for treating obesity." D. "With your permission, I'd like you to refer to a support group for individuals who live with severe obesity."

A. "It would be very helpful if you could integrate more physical activity into your routine."

The nurse is providing care for an adult client who has sought care for the treatment of obesity. When performing an assessment of this client, the nurse should address what potential contributing factors? Select all that apply. A. Activity level C. Family history and genetics D. Endocrine factors E. Microbiota

A. Activity level C. Family history and genetics D. Endocrine factors E. Microbiota

The nurse is caring for a client who has obesity and who has been prescribed naltrexone/bupropion. What assessments should the nurse perform? Select all that apply. A. Alcohol intake B. Mood and affect C. Cognition and orientation D. Skin integrity E. Blood pressure

A. Alcohol intake B. Mood and affect E. Blood pressure

The nurse is caring for a client that has undergone bariatric surgery. Which indication is a complication from the surgery? A. Dumping syndrome B. Cushing syndrome C. Malnutrition D. Diverticulitis

A. Dumping syndrome

A client has a recent diagnosis of chronic pancreatitis and is undergoing diagnostic testing to determine pancreatic islet cell function. The nurse should anticipate what diagnostic test? A. Glucose tolerance test B. ERCP C. Pancreatic biopsy D. Abdominal ultrasonography

A. Glucose tolerance test

A client with ongoing back pain, nausea, and abdominal bloating has been diagnosed with cholecystitis secondary to gallstones. The nurse should anticipate that the client will undergo what intervention? A. Laparoscopic cholecystectomy B. Methyl tertiary butyl ether (MTBE) infusion C. Intracorporeal lithotripsy D. Extracorporeal shock wave therapy (ESWL)

A. Laparoscopic cholecystectomy

A client with a cholelithiasis has been scheduled for a laparoscopic cholecystectomy. Why is laparoscopic cholecystectomy preferred by surgeons over an open procedure? A. Laparoscopic cholecystectomy poses fewer surgical risks than an open procedure. B. Laparoscopic cholecystectomy can be performed in a clinic setting, while an open procedure requires an OR. C. A laparoscopic approach allows for the removal of the entire gallbladder. D. A laparoscopic approach can be performed under conscious sedation.

A. Laparoscopic cholecystectomy poses fewer surgical risks than an open procedure.

An adult client has been admitted to the medical unit for the treatment of acute pancreatitis. What nursing action should be included in this client's plan of care? A. Measure the client's abdominal girth daily. B. Limit the use of opioid analgesics. C. Monitor the client for signs of dysphagia. D. Encourage activity as tolerated.

A. Measure the client's abdominal girth daily.

The nurse is admitting a client who has class III obesity. Inspection reveals that the client's neck and chest have a much larger than usual circumference. What is the nurse's best action? A. Perform a focused respiratory assessment. B. Assess for impaired skin integrity. C. Teach the client deep breathing and coughing exercises. D. Obtain a prescription for nebulized bronchodilators.

A. Perform a focused respiratory assessment.

The nurse is caring for a client who had an intragastric balloon placed 5 months ago for the treatment of obesity. The client's abdominal girth has increased over the past 48 hours and the last bowel movement was 72 hours ago. What is the nurse's best action? A. Report the possibility of balloon rupture to the primary provider. B. Monitor the client closely for nausea and vomiting. C. Document the therapeutic effects of the intragastric balloon. D. Encourage the client to consider having the balloon removed in the next 4 weeks.

A. Report the possibility of balloon rupture to the primary provider.

The nurse is assessing a client who has a chronic pain disorder and who also has class II obesity. What principle should guide the care team's choice of pain treatments for this client? A. The client may require higher doses of opioids than clients without obesity. B. The client is more likely to experience relief with NSAIDs than with opioids. C. The client's renal function must be monitored more closely during pain treatment than in clients without obesity. D. Adverse effects of opioids may be more difficult to assess than in clients without obesity.

A. The client may require higher doses of opioids than clients without obesity.

The nurse is assessing a client with obesity who has been taking naltrexone/bupropion for the past several weeks. What assessment finding most clearly suggests that the medication is having a desired effect? A. The client reports a diminished appetite and fewer cravings. B. The client is having one to two bowel movements daily, with fat present in stool. C. The client is losing at least 6 pounds (2.7 kg) per week, on average. D. The client is able to adhere to a low-carbohydrate, high-protein diet.

A. The client reports a diminished appetite and fewer cravings.

The nurse is caring for a hospitalized client who has class II obesity and who has limited mobility. The nurse should address the client's risk for skin breakdown by: A. cleaning and drying regularly within the client's skin folds. B. avoiding the use of pillows to position the client. C. making a referral to physical therapy. D. ensuring the client receives a high-calorie, high-protein diet

A. cleaning and drying regularly within the client's skin folds.

A nurse is assessing a patient who is experiencing peripheral neurovascular dysfunction. What assessment findings are most consistent with this diagnosis?

Absence of feeling, capillary refill of 4 to 5 seconds, and cool skin

A client's colorectal cancer has necessitated a hemicolectomy with the creation of a colostomy. In the 4 days since the surgery, the client has been unwilling to look at the ostomy or participate in any aspects of ostomy care. What is the nurse's most appropriate response to this observation? A. Ensure that the client knows that he or she will be responsible for care after discharge. B. Reassure the client that many people are fearful after the creation of an ostomy. C. Acknowledge the client's reluctance and initiate discussion of the factors underlying it. D. Arrange for the client to be seen by a social worker or spiritual advisor.

Acknowledge the client's reluctance and initiate discussion of the factors underlying it.

A nurse is performing a nursing assessment of a patient suspected of having a musculoskeletal disorder. What is the primary focus of the nursing assessment with a patient who has a musculoskeletal disorder?

Activities of daily living

A nurse is caring for a client who has an order to discontinue the administration of parenteral nutrition. What should the nurse do to prevent the occurrence of rebound hypoglycemia in the client? A. Administer an isotonic dextrose solution for 1 to 2 hours after discontinuing the PN. B. Administer a hypertonic dextrose solution for 1 to 2 hours after discontinuing the PN. C. Administer 3 ampules of dextrose 50% immediately prior to discontinuing the PN. D. Administer 3 ampules of dextrose 50% 1 hour after discontinuing the PN.

Administer an isotonic dextrose solution for 1 to 2 hours after discontinuing the PN.

The nurse is administering medications to a client through a feeding tube. Which action should the nurse take? A. Flush the tube with 5 mL of water before administering medication. B. Turn the tube feeding off for 1 hour before administering the medication. C. Administer each medication separately. D. Flush with 50 mL of water between each medication.

Administer each medication separately.

A community health nurse is preparing for an initial home visit to a client discharged following a total gastrectomy for treatment of gastric cancer. What would the nurse anticipate that the plan of care is most likely to include? A. Enteral feeding via gastrostomy tube (G tube) B. Gastrointestinal decompression by nasogastric tube C. Periodic assessment for esophageal distension D. Administration of injections of vitamin B12

Administration of injections of vitamin B12

A nurse is caring for a patient with a diagnosis of cancer that has metastasized. What laboratory value would the nurse expect to be elevated in this patient?

Alkaline phosphatase

A patient is undergoing diagnostic testing for suspected Pagets disease. What assessment finding is most consistent with this diagnosis?

Altered serum calcium levels

A client's health history is suggestive of inflammatory bowel disease. Which of the following would suggest Crohn disease, rather than ulcerative colitis, as the cause of the client's signs and symptoms? A. A pattern of distinct exacerbations and remissions B. Severe diarrhea C. An absence of blood in stool D. Involvement of the rectal mucosa

An absence of blood in stool

A nurse is providing care for a patient whose pattern of laboratory testing reveals longstanding hypocalcemia. What other laboratory result is most consistent with this finding?

An elevated parathyroid hormone level

A nurse is caring for a client with Paget disease and is reviewing the client's most recent laboratory values. Which of the following values are most characteristic of Paget disease?

An elevated serum alkaline phosphatase level and a normal serum calcium level

A 68-year-old client with a history of rheumatic disease has persistent swelling, no stiffness, and full range of motion to his left knee after an injury sustained several months ago. X-rays reveal no fracture of the extremity. Which factor is the most likely cause of the client's continued swelling?

An inflammation process

Which of the following clients should the nurse recognize as being at the highest risk for the development of osteomyelitis?

An older adult client with an infected pressure ulcer in the sacral area

The nurse is providing care for a client whose inflammatory bowel disease has necessitated hospital treatment. Which of the following would most likely be included in the client's medication regimen? A. Antidiarrheal medications 30 minutes before a meal B. Antiemetics on a PRN basis C. Vitamin B12 injections to prevent pernicious anemia D. Beta adrenergic blockers to reduce bowel motility

Antidiarrheal medications 30 minutes before a meal

A nurse is caring for a client with recurrent hematemesis who is scheduled for upper gastrointestinal fibroscopy. How should the nurse in the radiology department prepare this client? A. Insert a nasogastric tube. B. Administer a micro Fleet enema at least 3 hours before the procedure. C. Have the client lie in a supine position for the procedure. D. Apply local anesthetic to the back of the client's throat.

Apply local anesthetic to the back of the client's throat.

A client has suffered a muscle strain and is reporting pain at 6 on a 10-point scale. The nurse should recommend what action?

Applying a cold pack to the injured site

A nurse in a busy emergency department provides care for many clients who present with contusions, strains, or sprains. What are treatment modalities that are common to all of these musculoskeletal injuries? Select all that apply.

Applying ice Compression dressings Resting the affected extremity Elevating the injured limb

A client with rheumatoid arthritis comes into the clinic for a routine check-up. On assessment the nurse notes that the client appears to have lost some ability to function since the last office visit. What is the nurse's most appropriate action?

Arrange for the client to be assessed in the home environment.

A public health nurse is organizing a campaign that will address the leading cause of musculoskeletalrelated disability in the United States. The nurse should focus on what health problem?

Arthritis

A patient has been experiencing progressive increases in knee pain and diagnostic imaging reveals a worsening effusion in the synovial capsule. The nurse should anticipate which of the following?

Arthrocentesis

A client has recently been admitted to the orthopedic unit following total hip arthroplasty. The nurse assesses that the indwelling urinary catheter was removed one hour ago in the post-anesthesia care unit and that the client has not yet voided. Which action should the nurse take?

Ask if the client needs to void.

The nurse is preparing to check for tube placement in the client's stomach as well as measure the residual volume. What are these nursing actions attempting to prevent? A. Gastric ulcers B. Aspiration C. Abdominal distention D. Diarrhea

Aspiration

A nurse is assessing a client who has just been admitted to the postsurgical unit following surgical resection for the treatment of oropharyngeal cancer. What assessment should the nurse prioritize? A. Assess ability to clear oral secretions. B. Assess for signs of infection. C. Assess for a patent airway. D. Assess for ability to communicate.

Assess for a patent airway.

An older adult who resides in an assisted living facility has sought care from the nurse because of recurrent episodes of constipation. Which of the following actions should the nurse first perform? A. Encourage the client to take stool softener daily. B. Assess the client's food and fluid intake. C. Assess the client's surgical history. D. Encourage the client to take fiber supplements.

Assess the client's food and fluid intake.

A client's NG tube has become clogged after the nurse instilled a medication that was insufficiently crushed. The nurse has attempted to aspirate with a large-bore syringe, with no success. What should the nurse do next? A. Withdraw the NG tube 2 inches (5 cm) and reattempt aspiration. B. Attach a syringe filled with warm water and attempt an in-and-out motion of instilling and aspirating. C. Withdraw the NG tube slightly and attempt to dislodge by flicking the tube with the fingers. D. Remove the NG tube promptly and obtain an order for reinsertion from the primary care provider.

Attach a syringe filled with warm water and attempt an in-and-out motion of instilling and aspirating.

An older adult client has fallen in the home and is brought to the emergency department by ambulance with a suspected fractured hip. X-rays confirm a fracture of the left femoral neck. When planning assessments during the client's presurgical care, the nurse should be aware of the client's heightened risk of what complication?

Avascular necrosis

1) The nurse determines that a client who has undergone skin, tissue, and muscle grafting following a modified radical neck dissection requires suctioning. What is the nurse's priority when suctioning this client? A. Avoid applying suction on or near the suture line. B. Position client on the non-operative side with the head of the bed down. C. Assess the client's ability to perform self-suctioning. D. Evaluate the client's ability to swallow saliva and clear fluids.

Avoid applying suction on or near the suture line.

A client with a peptic ulcer disease has had metronidazole added to their current medication regimen. What health education related to this medication should the nurse provide? A. Take the medication on an empty stomach. B. Take up to one extra dose per day if stomach pain persists. C. Take at bedtime to mitigate the effects of drowsiness. D. Avoid drinking alcohol while taking the drug.

Avoid drinking alcohol while taking the drug.

A nurse is caring for an adult client diagnosed with a back strain. What health education should the nurse provide to this client?

Avoid lifting more than one-third of body weight without assistance.

A client will be undergoing a urea breath test for the detection of Helicobacter pylori. Which instruction should the nurse give to the client to prepare for this test? A. Ingest a capsule of carbon-labeled urea ingested three days before the test. B. Take prescribed antibiotics one month before the test. C. Fast for 12 hours before the test. D. Avoid taking cimetidine 24 hours before the test.

Avoid taking cimetidine 24 hours before the test.

A nurse is caring for a client who has just had a rigid fixation of a mandibular fracture. When planning the discharge teaching for this client, what would the nurse be sure to include? A. Increasing calcium intake to promote bone healing B. Avoiding chewing food for the specified number of weeks after surgery C. Techniques for managing parenteral nutrition in the home setting D. Techniques for managing a gastrostomy

Avoiding chewing food for the specified number of weeks after surgery

The nurse is providing care for a client who has recently been diagnosed with chronic gastritis. What health practice should the nurse address when teaching the client to limit exacerbations of the disease? A. Performing 15 minutes of physical activity at least three times per week B. Avoiding taking aspirin to treat pain or fever C. Taking multivitamins as prescribed and eating organic foods whenever possible D. Maintaining a healthy body weight

Avoiding taking aspirin to treat pain or fever

A client has presented to the emergency department with an injury to the wrist. The client is diagnosed with a third-degree strain. Why would the health care provider prescribe an x-ray of the wrist?

Avulsion fractures are associated with third-degree strains.

A client with a history of injection drug use has been diagnosed with hepatitis C. When collaborating with the care team to plan this client's treatment, the nurse should anticipate what intervention? A. Administration of immune globulins B. A regimen of antiviral medications C. Rest and watchful waiting D. Administration of fresh-frozen plasma (FFP)

B) A regimen of antiviral medications

A client has been diagnosed with advanced stage breast cancer and will soon begin aggressive treatment. What assessment findings would most strongly suggest that the client may have developed liver metastases? A. Persistent fever and cognitive changes B. Abdominal pain and hepatomegaly C. Peripheral edema unresponsive to diuresis D. Spontaneous bleeding and jaundice

B) Abdominal pain and hepatomegaly

What health promotion teaching should the nurse prioritize to prevent drug-induced hepatitis? A. Finish all prescribed courses of antibiotics, regardless of symptom resolution. B. Adhere to dosing recommendations of over-the-counter analgesics. C. Ensure that expired medications are disposed of safely. D. Ensure that pharmacists regularly review drug regimens for potential interactions.

B) Adhere to dosing recommendations of over-the-counter analgesics.

A client has had a laparoscopic cholecystectomy. The client is now reporting right shoulder pain. What should the nurse suggest to relieve the pain? A. Aspirin every 4 to 6 hours as prescribed B. Application of heat 15 to 20 minutes each hour C. Application of an ice pack for no more than 15 minutes D. Application of liniment rub to affected area

B) Application of heat 15 to 20 minutes each hour

A nurse is caring for a client who has been admitted for the treatment of advanced cirrhosis. What assessment should the nurse prioritize in this client's plan of care? A. Measurement of abdominal girth and body weight B. Assessment for variceal bleeding C. Assessment for signs and symptoms of jaundice D. Monitoring of results of liver function testing

B) Assessment for variceal bleeding

A client with cirrhosis has experienced a progressive decline in his health; and liver transplantation is being considered by the interdisciplinary team. How will the client's prioritization for receiving a donor liver be determined? A. By considering the client's age and prognosis B. By objectively determining the client's medical need C. By objectively assessing the client's willingness to adhere to post-transplantation care D. By systematically ruling out alternative treatment options

B) By objectively determining the client's medical need

A client with gallstones has been prescribed ursodeoxycholic acid (UDCA). The nurse understands that additional teaching is needed regarding this medication when the client states: A. "It is important that I see my health care provider for scheduled follow-up appointments while taking this medication." B. "I will take this medication for 2 weeks and then gradually stop taking it." C. "If I lose weight, the dose of the medication may need to be changed." D. "This medication will help dissolve small gallstones made of cholesterol."

B) I will take this medication for 2 weeks and then gradually stop taking it."

A nurse is preparing a plan of care for a client with pancreatic cysts that have necessitated drainage through the abdominal wall. What nursing diagnosis should the nurse prioritize? A. Disturbed body image B. Impaired skin integrity C. Nausea D. Risk for deficient fluid volume

B) Impaired skin integrity

A client with a diagnosis of esophageal varices has undergone endoscopy to gauge the progression of this complication of liver disease. Following the completion of this diagnostic test, what nursing intervention should the nurse perform? A. Keep client NPO until the results of test are known. B. Keep client NPO until the client's gag reflex returns. C. Administer analgesia until post-procedure tenderness is relieved. D. Give the client a cold beverage to promote swallowing ability.

B) Keep client NPO until the client's gag reflex returns.

A nurse is caring for a client with a blocked bile duct from a tumor. What manifestation of obstructive jaundice should the nurse anticipate? A. Watery, blood-streaked diarrhea B. Orange and foamy urine C. Increased abdominal girth D. Decreased cognition

B) Orange and foamy urine

A client with chronic pancreatitis had a pancreaticojejunostomy created 3 months ago for relief of pain and to restore drainage of pancreatic secretions. The client has come to the office for a routine postsurgical appointment. The client is frustrated that the pain has not decreased. What is the most appropriate initial response by the nurse? A. "The majority of clients who have a pancreaticojejunostomy have their normal digestion restored but do not achieve pain relief." B. "Pain relief occurs by 6 months in most clients who undergo this procedure, but some people experience a recurrence of their pain." C. "Your health care provider will likely want to discuss the removal of your gallbladder to achieve pain relief." D. "You are probably not appropriately taking the medications for your pancreatitis and pain, so we will need to discuss your medication regimen in detail."

B) Pain relief occurs by 6 months in most clients who undergo this procedure, but some people experience a recurrence of their pain."

A nurse is caring for a client with cirrhosis secondary to heavy alcohol use. The nurse's most recent assessment reveals subtle changes in the client's cognition and behavior. What is the nurse's most appropriate response? A. Ensure that the client's sodium intake does not exceed recommended levels. B. Report this finding to the primary provider due to the possibility of hepatic encephalopathy. C. Inform the primary provider that the client should be assessed for alcoholic hepatitis. D. Implement interventions aimed at ensuring a calm and therapeutic care environment.

B) Report this finding to the primary provider due to the possibility of hepatic encephalopathy.

A nurse is caring for a client who has been scheduled for endoscopic retrograde cholangiopancreatography (ERCP) the following day. When providing anticipatory guidance for this client, the nurse should describe what aspect of this diagnostic procedure? A. The need to protect the incision postprocedure B. The use of moderate sedation C. The need to infuse 50% dextrose during the procedure D. The use of general anesthesia

B) The use of moderate sedation

A client with obesity has been taking orlistat for the past several days. During the client's most recent follow-up assessment with the nurse, the client states, "I'm embarrassed to even say it, but I've had a few episodes of leaking stool since I've started this medication." What is the nurse's best response? A. "I'm sure that must be difficult for you. That's actually a sign that your body is breaking down fat tissue." B. "That sounds stressful for you. That's definitely one of the adverse effects of this medication." C. "Wearing an adult incontinence pad in the short term should resolve that problem for you." D. "Have you made any changes in your diet that might be contributing to this problem?"

B. "That sounds stressful for you. That's definitely one of the adverse effects of this medication."

A client has been admitted to the critical care unit with a diagnosis of toxic hepatitis. When planning the client's care, the nurse should be aware of what potential clinical course of this health problem? Place the following events in the correct sequence. 1. Fever rises. 2. Hematemesis. 3. Clotting abnormalities. 4. Vascular collapse. 5. Coma. A. 1, 2, 5, 4, 3 B. 1, 2, 3, 4, 5 C. 2, 3, 1, 4, 5 D. 3, 1, 2, 5, 4

B. 1, 2, 3, 4, 5

The nurse is providing care for a client who was recently diagnosed with chronic gastritis. What health practice should the nurse address when teaching the client to limit exacerbations of the disease? A. Perform 15 minutes of physical activity at least three times per week. B. Avoid taking aspirin to treat pain or fever. C. Take multivitamins as prescribed and eating organic foods whenever possible. D. Maintain a healthy body weight.

B. Avoid taking aspirin to treat pain or fever.

The nurse is caring for a client who has just returned from the ERCP removal of gallstones. The nurse should monitor the client for signs of what complications? A. Pain and peritonitis B. Bleeding and perforation C. Acidosis and hypoglycemia D. Gangrene of the gallbladder and hyperglyce

B. Bleeding and perforation

The community health nurse is leading a health education workshop for older adults. When addressing nutrition, what guideline should the nurse provide to participants? A. Clients should aim to get at least 30% of their caloric intake from fiber. B. Clients should aim to consume at least five servings of fruits or vegetables each day. C. Protein should make up around one third of daily calories. D. Clients should eliminate fat from their diet, if possible.

B. Clients should aim to consume at least five servings of fruits or vegetables each day.

40). An adult is postoperative day 2 following bariatric surgery. The nurse's most recent assessment reveals abdominal pain that the client rates at 8 out of 10, heart rate of 102 beats per minute and an oral temperature of 38.1°C (100.6°F). What is the nurse's best action? A. Slow the client's oral intake in consultation with the dietitian to prevent further dumping syndrome. B. Communicate the findings to the health care provider because the client may have an anastomotic leak. C. Insert a nasogastric tube as prescribed to facilitate STAT gastric decompression. D. Report the signs and symptoms of a possible surgical site infection to the health care provider.

B. Communicate the findings to the health care provider because the client may have an anastomotic leak.

A nurse is performing a health history on a client with obesity. Which condition is the client most at risk for with a diagnosis of obesity? A. Upper respiratory infections B. Diabetes C. Hypotension D. Pernicious anemia

B. Diabetes

A nurse is caring for a client after bariatric surgery and is assessing for hemorrhage. What is a sign of hemorrhage? A. Increase in blood pressure B. Frank red bleeding from the surgical site C. Clear drainage from the surgical wound D. Decrease in heart rate

B. Frank red bleeding from the surgical site

A client has been diagnosed with acute pancreatitis. The nurse is addressing the diagnosis of Acute Pain Related to Pancreatitis. What pharmacologic intervention is most likely to be ordered for this client? A. Oral oxycodone B. IV hydromorphone C. IM meperidine D. Oral naproxen

B. IV hydromorphone

The community health nurse is performing a home visit to a client who has obesity, peripheral vascular disease, and type 2 diabetes. The client has expressed a desire to lose weight. What is the nurse's best initial action? A. Teach the client about the relationship between lifestyle and body weight B. Identify the client's desired goals for weight loss C. Teach the client exercises that are physically achievable and easy to perform D. Review the client's most recent blood glucose and hemoglobin A1c results

B. Identify the client's desired goals for weight loss

The school nurse is working with a high school junior whose BMI is 31. When planning this client's care, the nurse should identify what goal? A. Continuation of current diet and activity level B. Increase in exercise and reduction in calorie intake C. Possible referral to an eating disorder clinic D. Increase in daily calorie intake

B. Increase in exercise and reduction in calorie intake

A nurse is caring for a client with gallstones who has been prescribed ursodeoxycholic acid (UDCA). The client asks how this medicine is going to help his symptoms. The nurse should be aware of what aspect of this drug's pharmacodynamics? A. It inhibits the synthesis of bile. B. It inhibits the synthesis and secretion of cholesterol. C. It inhibits the secretion of bile. D. It inhibits the synthesis and secretion of amylase.

B. It inhibits the synthesis and secretion of cholesterol.

A client with class II obesity has been unable to lose weight despite trying to increase activity and limit food intake. The health care provider has prescribed orlistat. What health education should the nurse provide to the client? A. The client will need to increase fluid intake during therapy. B. It is important to maintain a nutrient-rich diet and take multivitamins. C. The client will need to have blood levels of the medication drawn after 2 weeks. D. It is necessary to increase potassium intake and reduce sodium intake.

B. It is important to maintain a nutrient-rich diet and take multivitamins.

1) The nurse in the ICU is caring for a 47-year-old, obese male client who is in shock following a motor vehicle accident. What would be the main challenge in meeting this client's elevated energy requirements during prolonged rehabilitation? A. Loss of adipose tissue B. Loss of skeletal muscle C. Inability to convert adipose tissue to energy D. Inability to maintain normal body mass

B. Loss of skeletal muscle

The nurse is performing a health assessment of a client who has been taking antiobesity medications for several weeks. During the nurse's gastrointestinal assessment, the client reports bowel movements described as "greasy" and "oily." What medication is the client most likely taking? A. Lorcaserin B. Orlistat C. Liraglutide D. Phentermine

B. Orlistat

A client is admitted to the unit with acute cholecystitis. The health care provider has stated that surgery will be scheduled in 4 days. The client asks why the surgery is being put off for a week when he has a "sick gallbladder." What rationale would underlie the nurse's response? A. Surgery is delayed until the client can eat a regular diet without vomiting. B. Surgery is delayed until the acute symptoms subside. C. The client requires aggressive nutritional support prior to surgery. D. Time is needed to determine whether a laparoscopic procedure can be used.

B. Surgery is delayed until the acute symptoms subside.

An adult female client has a body mass index of 34.5 kg/m2 and has expressed interest in bariatric surgery. What characteristic of the client's health status may exclude her from being an appropriate surgical candidate? A. The client has poorly controlled type 2 diabetes B. The client drinks six to eight cans of beer daily C. The client smokes half a pack of cigarettes daily D. The client has a strong family history of obesity

B. The client drinks six to eight cans of beer daily

A client with obesity is early in the process of preparing for a Roux-en-Y gastric bypass (RYGB). The client states, "After the surgery, the amount of food that I consume will be limited and I'll absorb fewer calories from what I do eat." When responding to the client, the nurse should: A. explain that the surgery will not affect the absorption of nutrients. B. validate what the client understands about the surgical procedure. C. teach the client that RYGB does not restrict food intake. D. encourage the client to discuss the procedure with the surgeon.

B. validate what the client understands about the surgical procedure

A client is diagnosed with giant cell arteritis (GCA) and is placed on corticosteroids. A concern for this client is that the client will stop taking the medication as soon as the client starts to feel better. The nurse must emphasize the need for continued adherence to the prescribed medication so that the client can avoid which complication?

Blindness

A clinic nurse is caring for a patient with a history of osteoporosis. Which of the following diagnostic tests best allows the care team to assess the patients risk of fracture?

Bone densitometry

An older adult patient has symptoms of osteoporosis and is being assessed during her annual physical examination. The assessment shows that the patient will require further testing related to a possible exacerbation of her osteoporosis. The nurse should anticipate what diagnostic test?

Bone densitometry

An older, female client with osteoporosis has been hospitalized. Prior to discharge, when teaching the client, the nurse should include information about which major complication of osteoporosis?

Bone fracture

A client has a diagnosis of rheumatoid arthritis, and the primary provider has now prescribed cyclophosphamide. The nurse's subsequent assessments should address which potential adverse effect?

Bone marrow suppression

The nurse is caring for a client who had a low-profile gastrostomy device placed. Which instruction should the nurse give the client and family? A. Wear the tubing outside of clothing. B. Use tape to secure the device. C. Bring the connection tubing if going to the hospital. D. Change the wet-to-dry dressing daily.

Bring the connection tubing if going to the hospital.

A nurse who provides care in a community clinic assesses a wide range of individuals. The nurse should identify which client as having the highest risk for chronic pancreatitis? A. A 45-year-old obese woman with a high-fat diet B. An 18-year-old man who is a weekend binge drinker C. A 39-year-old man with chronic alcoholism D. A 51-year-old woman who smokes one-and-a-half packs of cigarettes per day

C) A 39-year-old man with chronic alcoholism

A nurse is caring for a client with severe hemolytic jaundice. Laboratory tests show free bilirubin to be 24 mg/dL (408 mmol/L). For what complication is this client at risk? A. Chronic jaundice B. Pigment stones in portal circulation C. Central nervous system damage D. Hepatomegaly

C) Central nervous system damage

A client with portal hypertension has been admitted to the medical floor. The nurse should prioritize what assessments? A. Assessment of blood pressure and assessment for headaches and visual changes B. Assessments for signs and symptoms of venous thromboembolism C. Daily weights and abdominal girth measurement D. Blood glucose monitoring q4h

C) Daily weights and abdominal girth measurement

A client with a diagnosis of cirrhosis has developed variceal bleeding and will imminently undergo variceal banding. What psychosocial nursing diagnosis should the nurse most likely prioritize during this phase of the client's treatment? A. Decisional conflict B. Deficient knowledge C. Death anxiety D. Disturbed thought processes

C) Death anxiety

A nurse has entered the room of a client with cirrhosis and found the client on the floor. The client reports falling when transferring to the commode. The client's vital signs are within reference ranges and the nurse observes no apparent injuries. What is the nurse's most appropriate action? A. Remove the client's commode and supply a bedpan. B. Complete an incident report and submit it to the unit supervisor. C. Have the client assessed by the primary provider due to the risk of internal bleeding. D. Perform a focused abdominal assessment in order to rule out injury.

C) Have the client assessed by the primary provider due to the risk of internal bleeding.

Diagnostic testing has revealed that a client's hepatocellular carcinoma (HCC) is limited to one lobe. The nurse should anticipate that this client's plan of care will focus on what intervention? A. Cryosurgery B. Liver transplantation C. Lobectomy D. Laser hyperthermia

C) Lobectomy

A nurse is performing an admission assessment of a client with a diagnosis of cirrhosis. What technique should the nurse use to palpate the client's liver? A. Place hand under the right lower abdominal quadrant and press down lightly with the other hand. B. Place the left hand over the abdomen and behind the left side at the 11th rib. C. Place hand under right lower rib cage and press down lightly with the other hand. D. Hold hand 90 degrees to right side of the abdomen and push down firmly.

C) Place hand under right lower rib cage and press down lightly with the other hand.

A client with liver disease has developed ascites; the nurse is collaborating with the client to develop a nutritional plan. The nurse should prioritize which of the following in the client's plan? A. Increased potassium intake B. Fluid restriction to 2 L per day C. Reduction in sodium intake D. High-protein, low-fat diet

C) Reduction in sodium intake

A client who had surgery for gallbladder disease has just returned to the postsurgical unit from postanesthetic recovery. The nurse caring for this client knows to immediately report what assessment finding to the health care provider? A. Decreased breath sounds B. Drainage of bile-colored fluid onto the abdominal dressing C. Rigidity of the abdomen D. Acute pain with movement

C) Rigidity of the abdomen

A client has undergone a laparoscopic cholecystectomy and is being prepared for discharge home. When providing health education, the nurse should prioritize what topic? A. Management of fluid balance in the home setting B. The need for blood glucose monitoring for the next week C. Signs and symptoms of intra-abdominal complications D. Appropriate use of prescribed pancreatic enzymes

C) Signs and symptoms of intra-abdominal complications

A nurse is caring for a client with hepatic encephalopathy. The nurse's assessment reveals that the client exhibits episodes of confusion, is difficult to arouse from sleep and has rigid extremities. Based on these clinical findings, the nurse should document what stage of hepatic encephalopathy? A. Stage 1 B. Stage 2 C. Stage 3 D. Stage 4

C) Stage 3

A client who has undergone liver transplantation is ready to be discharged home. Which outcome of health education should the nurse prioritize? A. The client will obtain measurement of drainage from the T-tube. B. The client will exercise three times a week. C. The client will take immunosuppressive agents as required. D. The client will monitor for signs of liver dysfunction.

C)The client will take immunosuppressive agents as required.

A diabetes educator is teaching a client about type 2 diabetes. The educator recognizes that the client understands the primary treatment for type 2 diabetes when the client states what? A. "I read that a pancreas transplant will provide a cure for my diabetes." B. "I will take my oral antidiabetic agents when my morning blood sugar is high." C. "I will make sure to follow the weight loss plan designed by the dietitian." D. "I will make sure I call the diabetes educator when I have questions about my insulin."

C. "I will make sure to follow the weight loss plan designed by the dietitian."

The nurse is working with a sedentary adult client who has expressed a determination to lose weight over the next several months, despite the presence of other major health problems. What is the nurse's best advice for this client? A. "We'll work together to ensure you don't exceed food intake of 2000 calories per day." B. "Try to perform both aerobic and muscle-training exercises every day." C. "It might be challenging to start an exercise program, but we'll start with a few minutes per day." D. "I'll make sure that you're screened for type 2 diabetes before you start your weight-loss program."

C. "It might be challenging to start an exercise program, but we'll start with a few minutes per day."

The nurse provides care for several clients who have obesity. Which client's obesity is most likely to resolve with medication? A. An obese client whose parents and siblings are not obese B. A client whose obesity is characterized as android rather than gynoid C. A client whose obesity has been attributed to hypothyroidism D. A client with long-standing obesity who has recently been diagnosed with type 2 diabetes

C. A client whose obesity has been attributed to hypothyroidism

A community health nurse is caring for a client whose multiple health problems include chronic pancreatitis. During the most recent home visit, the nurse learns that the client is experiencing severe abdominal pain and has vomited 3 times in the past several hours. What is the nurse's most appropriate action? A. Administer a PRN dose of pancreatic enzymes as prescribed. B. Teach the client about the importance of abstaining from alcohol. C. Arrange for the client to be transported to the hospital. D. Insert an NG tube, if available, and stay with the client.

C. Arrange for the client to be transported to the hospital.

A nurse is creating a care plan for a client with acute pancreatitis. The care plan includes reduced activity. What rationale for this intervention should be cited in the care plan? A. Bed rest reduces the client's metabolism and reduces the risk of metabolic acidosis. B. Reduced activity protects the physical integrity of pancreatic cells. C. Bed rest lowers the metabolic rate and reduces enzyme production. D. Inactivity reduces caloric need and gastrointestinal motility.

C. Bed rest lowers the metabolic rate and reduces enzyme production.

A student nurse is caring for a client who has a diagnosis of acute pancreatitis and who is receiving parenteral nutrition. The student should prioritize which of the following assessments? A. Fluid output B. Oral intake C. Blood glucose levels D. BUN and creatinine levels

C. Blood glucose levels

The nurse is providing an educational workshop about coronary artery disease (CAD) and its risk factors. The nurse explains to participants that CAD has many risk factors, some that can be controlled and some that cannot. What risk factors should the nurse list that can be controlled or modified? A. Gender, obesity, family history, and smoking B. Inactivity, stress, gender, and smoking C. Cholesterol levels, hypertension, and smoking D. Stress, family history, and obesity

C. Cholesterol levels, hypertension, and smoking

A client has just been diagnosed with chronic pancreatitis. The client is underweight and in severe pain and diagnostic testing indicates that over 80% of the client's pancreas has been destroyed. The client asks the nurse why the diagnosis was not made earlier in the disease process. What would be the nurse's best response? A. "The symptoms of pancreatitis mimic those of much less serious illnesses." B. "Your body doesn't require pancreatic function until it is under great stress, so it is easy to go unnoticed." C. "Chronic pancreatitis often goes undetected until a large majority of pancreatic function is lost." D. "It's likely that your other organs were compensating for your decreased pancreatic function."

C. Chronic pancreatitis often goes undetected until a large majority of pancreatic function is lost."

A client with obesity has expressed interest in bariatric surgery and has presented for an initial appointment at a bariatric clinic. The care team at the client should prioritize what action in the client's subsequent care? A. Encouraging the client to lose at least 2% of body weight by lifestyle means prior to surgery B. Encouraging the client to lose at least 4% of body weight by lifestyle means prior to surgery C. Determining whether the client is an appropriate candidate for surgery D. Screening the client for exclusions to bariatric surgery such as frequent cycles of weight loss and weight gain

C. Determining whether the client is an appropriate candidate for surgery

A client has been admitted to the hospital for the treatment of chronic pancreatitis. The client has been stabilized and the nurse is now planning health promotion and educational interventions. Which of the following should the nurse prioritize? A. Educating the client about expectations and care following surgery B. Educating the client about the management of blood glucose after discharge C. Educating the client about postdischarge lifestyle modifications D. Educating the client about the potential benefits of pancreatic transplantation

C. Educating the client about postdischarge lifestyle modifications

A client is receiving care in the intensive care unit for acute pancreatitis. The nurse is aware that pancreatic necrosis is a major cause of morbidity and mortality in clients with acute pancreatitis. Consequently, the nurse should assess for what signs or symptoms of this complication? A. Sudden increase in random blood glucose readings B. Increased abdominal girth accompanied by decreased level of consciousness C. Fever, increased heart rate and decreased blood pressure D. Abdominal pain unresponsive to analgesics

C. Fever, increased heart rate and decreased blood pressure

A nurse is providing discharge education to a client who has undergone a laparoscopic cholecystectomy. During the immediate recovery period, the nurse should recommend what foods? A. High-fiber foods B. Low-purine, nutrient-dense foods C. Low-fat foods high in proteins and carbohydrates D. Foods that are low-residue and low in fat

C. Low-fat foods high in proteins and carbohydrates

The nurse is caring for a client who had bariatric surgery several months ago and who has been prescribed monthly vitamin B12 injections. How should the nurse best assess for the therapeutic effect of these injections? A. Monitor the client's body mass index (BMI) weekly. B. Monitor the client's hemoglobin and hematocrit results. C. Measure the client's abdominal girth and review electrolyte levels. D. Assess the client's skin integrity and nutritional status.

C. Measure the client's abdominal girth and review electrolyte levels.

A 45-year-old obese man arrives in a clinic reporting daytime sleepiness, difficulty going to sleep at night, and snoring. The nurse should recognize the manifestations of what health problem? A. Adenoiditis B. Chronic tonsillitis C. Obstructive sleep apnea D. Laryngeal cancer

C. Obstructive sleep apnea

A client is admitted to the ICU with acute pancreatitis. The client's family asks what causes acute pancreatitis. The critical care nurse knows that a majority of clients with acute pancreatitis have what health issue? A. Type 1 diabetes B. An impaired immune system C. Undiagnosed chronic pancreatitis D. An amylase deficiency

C. Undiagnosed chronic pancreatitis

A nurse is reviewing the pathophysiology that may underlie a client's decreased bone density. What hormone should the nurse identify as inhibiting bone resorption and promoting bone formation?

Calcitonin

The nurse is assessing a patient for dietary factors that may influence her risk for osteoporosis. The nurse should question the patient about her intake of what nutrients? Select all that apply.

Calcium Vitamin D

A client tells the nurse that they haves pain and numbness in the thumb, first finger, and second finger of the right hand. The nurse discovers that the client is employed as an auto mechanic, and that the pain is increased while working. This may indicate that the client has what health problem?

Carpel tunnel syndrome

A nurse is teaching a client with osteomalacia about the role of diet. What would be the best choice for breakfast for a client with osteomalacia?

Cereal with milk, a scrambled egg, and grapefruit

A nurse is aware of the high incidence of catheter-related bloodstream infections in clients receiving parenteral nutrition. What nursing action has the greatest potential to reduce catheter-related bloodstream infections? A. Use clean technique and wear a mask during dressing changes. B. Change the dressing no more than weekly. C. Apply antibiotic ointment around the site with each dressing change. D. Irrigate the insertion site with sterile water during each dressing change.

Change the dressing no more than weekly.

The nurse is administering total parenteral nutrition (TPN) to a client who underwent surgery for gastric cancer. Which of the nurse's assessments most directly addresses a major complication of TPN? A. Checking the client's capillary blood glucose levels regularly B. Having the client frequently rate his or her hunger on a 10-point scale C. Measuring the client's heart rhythm at least every 6 hours D. Monitoring the client's level of consciousness each shift

Checking the client's capillary blood glucose levels regularly

A nurse is caring for an older adult client who is preparing for discharge following recovery from a total hip replacement. What outcome must be met prior to discharge?

Client is able to perform transfers safely.

A nurse is providing care for a client who has a diagnosis of irritable bowel syndrome (IBS). When planning this client's care, the nurse should collaborate with the client and prioritize what goal? A. Client will accurately identify foods that trigger symptoms. B. Client will demonstrate appropriate care of his ileostomy. C. Client will demonstrate appropriate use of standard infection control precautions. D. Client will adhere to recommended guidelines for mobility and activity.

Client will accurately identify foods that trigger symptoms.

A nurse is providing discharge teaching for a client who underwent foot surgery. The nurse is collaborating with the occupational therapist and discussing the use of assistive devices. On what variables does the choice of assistive devices primarily depend?

Client's general condition, balance, and weight-bearing prescription

While assessing a patient, the patient tells the nurse that she is experiencing rhythmic muscle contractions when the nurse performs passive extension of her wrist. What is this pattern of muscle contraction referred to as?

Clonus

A client has come to the outpatient radiology department for diagnostic testing that will allow the care team to evaluate and remove polyps. The nurse should prepare the client for what procedure? A. Colonoscopy B. Barium enema C. ERCP D. Upper gastrointestinal fibroscopy

Colonoscopy

Radiographs of a client's upper arm shows three fragments of the humeral bone. This diagnostic result suggests what type of fracture?

Comminuted

The nurse is performing an assessment of a patients musculoskeletal system and is appraising the patients bone integrity. What action should the nurse perform during this phase of assessment?

Compare parts of the body symmetrically.

A nurse admits a client who has a fracture of the nose that has resulted in a skin tear and involvement of the mucous membranes of the nasal passages. The orthopedic nurse should plan to care for what type of fracture?

Compound

A client has been admitted to the hospital with a spontaneous vertebral fracture related to osteoporosis. Which of the following nursing diagnoses must be addressed in the plan of care?

Constipation related to vertebral fracture

1) A nurse is working with a client who has chronic constipation. What should be included in client teaching to promote normal bowel function? A. Use glycerin suppositories on a regular basis. B. Limit physical activity in order to promote bowel peristalsis. C. Consume high-residue, high-fiber foods. D. Resist the urge to defecate until the urge becomes intense.

Consume high-residue, high-fiber foods.

A patient has had a cast placed for the treatment of a humeral fracture. The nurses most recent assessment shows signs and symptoms of compartment syndrome. What is the nurses most appropriate action?

Contact the primary care provider immediately.

A client admitted with acute diverticulitis has experienced a sudden increase in temperature and reports a sudden onset of exquisite abdominal tenderness. The nurse's rapid assessment reveals that the client's abdomen is uncharacteristically rigid on palpation. What is the nurse's best response? A. Administer a Fleet enema as prescribed and remain with the client. B. Contact the primary care provider promptly and report these signs of perforation. C. Position the client supine and insert an NG tube. D. Page the primary provider and report that the client may be obstructed.

Contact the primary care provider promptly and report these signs of perforation.

A patient injured in a motor vehicle accident has sustained a fracture to the diaphysis of the right femur. Of what is the diaphysis of the femur mainly constructed?

Cortical bone

A nurse is performing a musculoskeletal assessment of a patient with arthritis. During passive range-ofmotion exercises, the nurse hears an audible grating sound. The nurse should document the presence of which of the following?

Crepitus

A nurse is reviewing the care of a client who has a long history of lower back pain that has not responded to conservative treatment measures. The nurse should anticipate the administration of what drug?

Cyclobenzaprine

A client has been newly diagnosed with acute pancreatitis and admitted to the acute medical unit. How should the nurse explain the pathophysiology of this client's health problem? A. "Toxins have accumulated and inflamed your pancreas." B. "Bacteria likely migrated from your intestines and became lodged in your pancreas." C. "A virus that was likely already present in your body has begun to attack your pancreatic cells." D. "The enzymes that your pancreas produces have damaged the pancreas itself."

D) "The enzymes that your pancreas produces have damaged the pancreas itself."

A nurse is performing an admission assessment for an 81-year-old client who generally enjoys good health. When considering normal, age-related changes to hepatic function, the nurse should anticipate what finding? A. Similar liver size and texture as in younger adults B. A nonpalpable liver C. A slightly enlarged liver with palpably hard edges D. A slightly decreased size of the liver

D) A slightly decreased size of the liver

A client is being treated for acute pain from an episode of pancreatitis. The nurse has identified a nursing diagnosis of Ineffective Breathing Pattern related to pain secondary to effects of surgery. Which intervention should the nurse perform in order to best address this diagnosis? A. Position the client supine to facilitate diaphragm movement. B. Administer corticosteroids by nebulizer as prescribed. C. Perform oral suctioning as needed to remove secretions. D. Administer analgesic per orders.

D) Administer analgesic per orders.

A client with esophageal varices is being cared for in the ICU. The varices have begun to bleed. The client has Ringer lactate at 150 cc/hr infusing. The nurse should also anticipate what intervention? A. Positioning the client supine B. Administering diuretics C. Oxygen by nasal cannula D. Administering volume expanders

D) Administering volume expanders

A nurse is amending a client's plan of care in light of the fact that the client has recently developed ascites. What should the nurse include in this client's care plan? A. Mobilization with assistance at least 4 times daily B. Administration of beta-adrenergic blockers as prescribed C. Vitamin B12 injections as prescribed D. Administration of diuretics as prescribed

D) Administration of diuretics as prescribed

A local public health nurse is informed that a cook in a local restaurant has been diagnosed with hepatitis A. What should the nurse advise individuals to obtain who ate at this restaurant and have never received the hepatitis A vaccine? A. The hepatitis A vaccine B. Albumin infusion C. The hepatitis A and B vaccines D. An immune globulin injection

D) An immune globulin injection

A client with liver cancer is being discharged home with a biliary drainage system in place. The nurse should teach the client's family how to safely perform which of the following actions? A. Aspirating bile from the catheter using a syringe B. Removing the catheter when output is 15 mL in 24 hours C. Instilling antibiotics into the catheter D. Assessing the patency of the drainage catheter

D) Assessing the patency of the drainage catheter

A client returns to the floor after a laparoscopic cholecystectomy. The nurse should assess the client for signs and symptoms of what serious potential complication of this surgery? A. Diabetic coma B. Decubitus ulcer C. Wound evisceration D. Bile duct injury

D) Bile duct injury

A client with liver cancer is being discharged home with a hepatic artery catheter in place. The nurse should be aware that this catheter will facilitate which of the following? A. Continuous monitoring for portal hypertension B. Administration of immunosuppressive drugs during the first weeks after transplantation C. Real-time monitoring of vascular changes in the hepatic system D. Delivery of a continuous chemotherapeutic dose

D) Delivery of a continuous chemotherapeutic dose

A client has been treated in the hospital for an episode of acute pancreatitis. The client has acknowledged the role that his alcohol use played in the development of his health problem, but has not expressed specific plans for lifestyle changes. What is the nurse's most appropriate response? A. Educate the client about the link between alcohol use and pancreatitis. B. Ensure that the client knows the importance of attending follow-up appointments. C. Refer the client to social work or spiritual care. D. Encourage the client to connect with a community-based support group.

D) Encourage the client to connect with a community-based support group.

A nurse is participating in the emergency care of a client who has just developed variceal bleeding. What intervention should the nurse anticipate? A. Infusion of intravenous heparin B. IV administration of albumin C. STAT administration of vitamin K by the intramuscular route D. IV administration of octreotide

D) IV administration of octreotide

1) A nurse is caring for a client with liver failure and is performing an assessment of the client's increased risk of bleeding. The nurse recognizes that this risk is related to the client's inability to synthesize prothrombin in the liver. What factor most likely contributes to this loss of function? A. Alterations in glucose metabolism B. Retention of bile salts C. Inadequate production of albumin by hepatocytes D. Inability of the liver to use vitamin K

D) Inability of the liver to use vitamin K

A home health nurse is caring for a client discharged home after pancreatic surgery. The nurse documents the nursing diagnosis Risk for Imbalanced Nutrition: Less than Body Requirements on the care plan based on the potential complications that may occur after surgery. What are the most likely complications for the client who has had pancreatic surgery? A. Proteinuria and hyperkalemia B. Hemorrhage and hypercalcemia C. Weight loss and hypoglycemia D. Malabsorption and hyperglycem

D) Malabsorption and hyperglycemia

1) A nurse is assessing a client who has been diagnosed with cholecystitis, and is experiencing localized abdominal pain. When assessing the characteristics of the client's pain, the nurse should anticipate that it may radiate to what region? A. Left upper chest B. Inguinal region C. Neck or jaw D. Right shoulder

D) Right shoulder

A nurse is assessing an older adult client with gallstones. The nurse is aware that the client may not exhibit typical symptoms, and that particular symptoms that may be exhibited in the elderly client may include what examples? A. Fever and pain B. Chills and jaundice C. Nausea and vomiting D. Signs and symptoms of septic shock

D) Signs and symptoms of septic shock

The nurse is speaking with a 20-year-old client who has class II obesity. The client states, "No matter what I do, I can't lose weight. How soon do you think I could get bariatric surgery?" What is the nurse's best response? A. "In most cases, clients have to be older than 25 in order to be candidates for bariatric surgery." B. "Depending on the results of your assessments, you can likely have surgery very soon." C. "Bariatric surgery does help many clients lose weight, but it's normally reserved for clients with more severe obesity than yours." D. "Bariatric surgery is often very effective, but it usually happens after a lengthy process of consultations and referrals."

D. "Bariatric surgery is often very effective, but it usually happens after a lengthy process of consultations and referrals."

A client with obesity has recently begun treatment with phentermine/topiramate-ER. The client tells the nurse, "I'm eating a lot of spinach and other leafy green vegetables, both cooked and in salads." What is the nurse's best response? A. "Spinach is very healthy, but eating it too often can be hard on your kidneys." B. "That's a healthy practice, but you might find that your blood clots more slowly than usual." C. "Be careful that you don't eat too many other foods that contain a lot of iron." D. "That's great. Spinach has a lot of vitamins and nutrients and very few calories."

D. "That's great. Spinach has a lot of vitamins and nutrients and very few calories."

A client with obesity has expressed a desire to lose weight and states that his previous efforts have been unsuccessful. The client states that he would like to sign up for a commercial weight-loss program. What is the nurse's best response? A. "Commercial weight-loss programs have been shown to be ineffective." B. "The majority of commercial programs don't work, and a large proportion of them are actually dangerous." C. "It would be unethical for me to recommend or warn against a commercial weight-loss program." D. "We can help you make a plan that wouldn't require you to join a commercial program."

D. "We can help you make a plan that wouldn't require you to join a commercial program."

A 69-year-old client has maintained a consistent diet and activity level throughout adulthood. Over the past few years, however, the client has reported a gradual increase in adipose tissue. When providing health education, the nurse should address what topic? A. Weight gain as a natural, age-related change B. Loss of skeletal muscle with aging C. Changes in food cravings that are common in older adults D. Changes in metabolism that accompany the aging procesS

D. Changes in metabolism that accompany the aging proces

A client presents to the emergency department (ED) reporting severe right upper quadrant pain. The client states that the family doctor said the pain was caused by gallstones. The ED nurse should recognize what possible complication of gallstones? A. Acute pancreatitis B. Atrophy of the gallbladder C. Gallbladder cancer D. Gangrene of the gallbladder

D. Gangrene of the gallbladder

A client has been diagnosed with pancreatic cancer and has been admitted for care. Following initial treatment, the nurse should be aware that the client is most likely to require which of the following situations? A. Inpatient rehabilitation B. Rehabilitation in the home setting C. Intensive physical therapy D. Hospice care

D. Hospice care

A client has recently been diagnosed with type 2 diabetes. The client is clinically obese and has a sedentary lifestyle. How can the nurse best begin to help increase the client's activity level? A. Set up appointment times at a local fitness center for the client to attend. B. Have a family member ensure the client follows a suggested exercise plan. C. Construct an exercise program and have the client follow it. D. Identify barriers with the client that inhibit his lifestyle change.

D. Identify barriers with the client that inhibit his lifestyle change.

The nurse is creating the care plan for a 70-year-old obese client who has been admitted to the postsurgical unit following a colon resection. This client's age and increased body mass index mean that the client is at increased risk for what complication in the postoperative period? A. Hyperglycemia B. Azotemia C. Falls D. Infection

D. Infection

. The nurse is assessing a hospital client who has severe obesity. The nurse's review of the client's latest arterial blood gases reveals PaCO2 of 48 mm Hg and PaO2 of 76 mm Hg. Inspection reveals a respiratory rate of 22 breaths per minute with shallow ventilation. What intervention should be included in the client's plan of care? A. Keep physical activity to a minimum. B. Keep the head of the client's bed at a 90-degree angle. C. Ambulate the client at least every 2 hours. D. Keep the client in a low Fowler position when in bed.

D. Keep the client in a low Fowler position when in bed.

A school nurse is teaching a group of high school students about risk factors for diabetes. What action has the greatest potential to reduce an individual's risk for developing diabetes? A. Have blood glucose levels checked annually. B. Stop using tobacco in any form. C. Undergo eye examinations regularly. D. Lose weight, if obese.

D. Lose weight, if obese.

The nurse is providing care for an adult client who has expressed frustration at the inability to lose weight, despite trying to reduce food intake. What aspect of this client's current health status should the nurse address? A. The client's spouse is also trying to lose weight at the same time. B. The client has a history of gastroesophageal reflux disease. C. The client takes levothyroxine for the treatment of hypothyroidism. D. The client takes a tricyclic antidepressant and has done so for several years

D. The client takes a tricyclic antidepressant and has done so for several years

A client with obesity has been prescribed liraglutide by the primary provider. When providing the client with health education, the nurse should teach the client: A. that the medication should be taken 30 minutes before each meal. B. about the need to avoid grapefruit and grapefruit juice. C. the signs and symptoms of acute kidney injury. D. how to self-administer subcutaneous injections.

D. how to self-administer subcutaneous injections.

A nurse is performing an abdominal assessment of an older adult client. When collecting and analyzing data, the nurse should be cognizant of what age-related change in gastrointestinal structure and function? A. Increased gastric motility B. Decreased gastric pH C. Increased gag reflex D. Decreased mucus secretion

Decreased mucus secretion

Allopurinol has been prescribed for a client receiving treatment for gout. The nurse caring for this client knows to assess the client for bone marrow suppression, which may be manifested by what diagnostic finding?

Decreased platelets

A client is admitted to the orthopedic unit with a fractured femur after a motorcycle accident. The client has been placed in traction until the femur can be rodded in surgery. For what early complication(s) should the nurse monitor this client? Select all that apply.

Deep vein thrombosis Compartment syndrome Fat embolism

A client has returned to the unit after undergoing limb-sparing surgery to remove a metastatic bone tumor. The nurse providing postoperative care in the days following surgery assesses for what complication from surgery?

Delayed wound healing

A client's electronic health record notes that the client has hallux valgus. What signs and symptoms should the nurse expect this client to manifest?

Deviation of a great toe laterally

A client is receiving education about an upcoming Billroth I procedure (gastroduodenostomy). This client should be informed that the client may experience which of the following adverse effects associated with this procedure? A. Persistent feelings of hunger and thirst B. Constipation or bowel incontinence C. Diarrhea and feelings of fullness D. Gastric reflux and belching

Diarrhea and feelings of fullness

35) A client with gastric cancer has been scheduled for a total gastrectomy. During the preoperative assessment, the client confides in the nurse feeling the surgery will "mutilate" the client's body. The nurse should plan interventions that address what nursing diagnosis? A. Disturbed body image B. Deficient knowledge related to the risks of surgery C. Anxiety about the surgery D. Low self-esteem

Disturbed body image

A nurse is assessing a client's stoma on postoperative day 3. The nurse notes that the stoma has a shiny appearance and a bright red color. How should the nurse best respond to this assessment finding? A. Irrigate the ostomy to clear a possible obstruction. B. Contact the primary care provider to report this finding. C. Document that the stoma appears healthy and well perfused. D. Document a nursing diagnosis of Impaired Skin Integrity.

Document that the stoma appears healthy and well perfused.

The nurse is assessing a client who had an ileostomy created three days ago for the treatment of irritable bowel disease. The nurse observes that the client's stoma is bright red and there are scant amounts of blood on the stoma. What is the nurse's best action? A. Contact the care provider to have the client's hemoglobin and hematocrit measured. B. Document these expected assessment findings. C. Apply barrier ointment to the stoma as prescribed. D. Cleanse the stoma with alcohol or chlorhexidine.

Document these expected assessment findings.

A client has been prescribed cimetidine for the treatment of peptic ulcer disease. When providing relevant health education for this client, the nurse should ensure the client is aware of what potential outcome? A. Bowel incontinence B. Drug-drug interactions C. Abdominal pain D. Heat intolerance

Drug-drug interaction

A client presents at the clinic with a report of morning numbness, cramping, and stiffness in the fourth and fifth fingers of the right hand. What disease process should the nurse suspect?

Dupuytren disease

A nurse is addressing the prevention of esophageal cancer in response to a question posed by a participant in a health promotion workshop. What action should the nurse recommend as having the greatest potential to prevent esophageal cancer? A. Promotion of a nutrient-dense, low-fat diet B. Annual screening endoscopy for clients over 50 with a family history of esophageal cancer C. Early diagnosis and treatment of gastroesophageal reflux disease D. Adequate fluid intake and avoidance of spicy foods

Early diagnosis and treatment of gastroesophageal reflux disease

A The client is experiencing painful oral lesions following radiation for oropharyngeal cancer. Which instruction should the nurse give this client? A. Spicy foods stimulate salivation and are soothing. B. Eat food while it is hot to enhance flavor. C. Avoid brushing teeth while lesions are present. D. Eat soft or liquid foods.

Eat soft or liquid foods.

A nurse is caring for a client who is 12 hours' postoperative following foot surgery. The nurse assesses the presence of edema in the foot. What nursing measure should the nurse implement to control the edema?

Elevate the foot on several pillows.

A patient is scheduled for a bone scan to rule out osteosarcoma of the pelvic bones. What would be most important for the nurse to assess before the patients scan?

Empty bladder

A nurse is caring for a client who is postoperative from a neck dissection. What would be the most appropriate nursing action to enhance the client's appetite? A. Encourage the family to bring in the client's favorite foods. B. Limit visitors at mealtimes so that the client is not distracted. C. Avoid offering food unless the client initiates. D. Provide thorough oral care immediately after the client eats.

Encourage the family to bring in the client's favorite foods.

A client with a total hip replacement has developed decreased breath sounds What is the nurse's best action?

Encourage use of the incentive spirometer.

A nurse is caring for a client who had a right below-the-knee amputation (BKA). The nurse recognizes the importance of implementing measures that focus on preventing flexion contracture of the hip and maintaining proper positioning. What nursing action will best achieve these goals?

Encouraging the client to turn from side to side and to assume a prone position

A client with a simple arm fracture is receiving discharge education from the nurse. What would the nurse instruct the client to do?

Engage in exercises that strengthen the unaffected muscles.

A nurse is providing care for a client whose recent colostomy has contributed to a nursing diagnosis of Disturbed Body Image Related to Colostomy. What intervention best addresses this diagnosis? A. Encourage the client to conduct online research into colostomies. B. Engage the client in dialogue about the implications of having the colostomy. C. Emphasize the fact that the colostomy was needed to alleviate a much more serious health problem. D. Emphasize the fact that the colostomy is temporary measure and is not permanent.

Engage the client in dialogue about the implications of having the colostomy.

A nurse is caring for a client who has had surgery for oral cancer. When addressing the client's long-term needs, the nurse should prioritize interventions and referrals with what goal? A. Enhancement of verbal communication B. Enhancement of immune function C. Maintenance of adequate social support D. Maintenance of fluid balance

Enhancement of verbal communication

A nurse is planning the care of an older adult client with osteomalacia. What action should the nurse recommend in order to promote vitamin D synthesis?

Ensuring adequate exposure to sunlight

A nurse is caring for a patient who has an MRI scheduled. What is the priority safety action prior to this diagnostic procedure?

Ensuring that there are no metal objects on or in the patient

1) A nurse is caring for a client who just has been diagnosed with a peptic ulcer. When teaching the client about his new diagnosis, how should the nurse best describe it? A. Inflammation of the lining of the stomach B. Erosion of the lining of the stomach or intestine C. Bleeding from the mucosa in the stomach D. Viral invasion of the stomach wall

Erosion of the lining of the stomach or intestine

A nurse is performing the initial assessment of a client who has a recent diagnosis of systemic lupus erythematosus (SLE). Which skin manifestation would the nurse expect to observe on inspection?

Erythematous rash

A nurse is caring for a client who is suspected of having giant cell arteritis (GCA). Which laboratory tests are most useful in diagnosing this rheumatic disorder?

Erythrocyte sedimentation rate C-reactive protein

A client was treated in the emergency department and critical care unit after ingesting bleach. What possible complication of the resulting gastritis should the nurse recognize? A. Esophageal or pyloric obstruction related to scarring B. Uncontrolled proliferation of H. pylori C. Gastric hyperacidity related to excessive gastrin secretion D. Chronic referred pain in the lower abdome

Esophageal or pyloric obstruction related to scarring

A nurse is taking a health history on a patient with musculoskeletal dysfunction. What is the primary focus of this phase of the nurses assessment?

Evaluating the effects of the musculoskeletal disorder on the patients function

The nurse is providing care for a client who has had a below-the-knee amputation. The nurse enters the client's room and finds the client resting in bed with the residual limb supported on a pillow. What is the nurse's most appropriate action?

Explain the risks of flexion contracture to the client.

A client has just begun been receiving skeletal traction and the nurse is aware that muscles in the client's affected limb are spastic. How does this change in muscle tone affect the client's traction prescription?

Extra weight is needed initially to keep the limb in proper alignment.

A client with polymyositis is experiencing challenges with activities of daily living as a result of proximal muscle weakness. What is the most appropriate nursing action?

Facilitate referrals to occupational and physical therapy

A nurse is providing anticipatory guidance to a client who is preparing for a total gastrectomy. The nurse learns that the client is anxious about numerous aspects of the surgery. What intervention is most appropriate to alleviate the client's anxiety? A. Emphasize the fact that gastric surgery has a low risk of complications. B. Encourage the client to focus on the benefits of the surgery. C. Facilitate the client's contact with support services. D. Obtain an order for a PRN benzodiazepine.

Facilitate the clients contact with support services.

The nurses musculoskeletal assessment of a patient reveals involuntary twitching of muscle groups. How would the nurse document this observation in the patients chart?

Fasciculations

A clinic nurse is caring for a client newly diagnosed with fibromyalgia. When developing a care plan for this client, which nursing diagnosis should the nurse prioritize?

Fatigue related to pain

A nurse is caring for a patient who has just had an arthroscopy as an outpatient and is getting ready to go home. The nurse should teach the patient to monitor closely for what postprocedure complication?

Fever

The human body is designed to protect its vital parts. A fracture of what type of bone may interfere with the protection of vital organs?

Flat bones

An adult client is scheduled for an upper GI series that will use a barium swallow. What teaching should the nurse include when the client has completed the test? A. Stool will be yellow for the first 24 hours' postprocedure. B. The barium may cause diarrhea for the next 24 hours. C. Fluids must be increased to facilitate the evacuation of the stool. D. Slight anal bleeding may be noted as the barium is passed.

Fluids must be increased to facilitate the evacuation of the stool.

Diagnostic tests show that a patients bone density has decreased over the past several years. The patient asks the nurse what factors contribute to bone density decreasing. What would be the nurses best response?

For many people, lack of nutrition can cause a loss of bone density.

The nurse is caring for a client who is undergoing diagnostic testing for suspected malabsorption. When taking this client's health history and performing the physical assessment, the nurse should recognize what finding as most consistent with this diagnosis? A. Recurrent constipation coupled with weight loss B. Foul-smelling diarrhea that contains fat C. Fever accompanied by a rigid, tender abdomen D. Bloody bowel movements accompanied by fecal incontinence

Foul-smelling diarrhea that contains fat

A client with a diagnosis of colon cancer is 2 days' postoperative following bowel resection and anastomosis. The nurse has planned the client's care in the knowledge of potential complications. What assessment should the nurse prioritize? A. Close monitoring of temperature B. Frequent abdominal auscultation C. Assessment of hemoglobin, hematocrit, and red blood cell levels D. Palpation of peripheral pulses and leg girth

Frequent abdominal auscultation

A client has been experiencing disconcerting GI symptoms that have been worsening in severity. Following medical assessment, the client has been diagnosed with lactose intolerance. The nurse should recognize an increased need for what form of health promotion? A. Annual screening colonoscopies B. Adherence to recommended immunization schedules C. Regular blood pressure monitoring D. Frequent screening for osteoporosis

Frequent screening for osteoporosis

A community health nurse is performing a visit to the home of a client who has a history of rheumatoid arthritis (RA). On which aspect of the client's health should the nurse focus most closely during the visit?

Functional status

A nurse is assessing the abdomen of a client just admitted to the unit with suspected GI disease. Inspection reveals several diverse lesions on the client's abdomen. How should the nurse best interpret this assessment finding? A. Abdominal lesions are usually due to age-related skin changes. B. Integumentary diseases often cause GI disorders. C. GI diseases often produce skin changes. D. The client needs to be assessed for self-harm.

GI diseases often produce skin changes

A client who was just diagnosed with scleroderma will be undergoing tests to assess for systemic involvement. Which system should the nurse prioritize in assessment?

Gastrointestinal

A client presents at a clinic reports heel pain that impairs walking ability. The client is subsequently diagnosed with plantar fasciitis. This client's plan of care should include what intervention?

Gently stretching the foot and the Achilles tendon

An emergency department nurse is admitting a 3-year-old brought in after swallowing a piece from a wooden puzzle. The nurse should anticipate the administration of what medication in order to relax the esophagus to facilitate removal of the foreign body? A. Haloperidol B. Prostigmine C. Epinephrine D. Glucagon

Glucagon

A nurse is providing care for a client who has a recent diagnosis of giant cell arteritis (GCA). Which aspect of physical assessment should the nurse prioritize?

Headaches and jaw pain

A client asks the nursing assistant for a bedpan. When the client is finished, the nursing assistant notifies the nurse that the client has bright red streaking of blood in the stool. The nurse's assessment should focus on what potential cause? A. Diet high in red meat B. Upper GI bleed C. Hemorrhoids D. Use of iron supplements

Hemorrhoids

A clinic client has described recent dark-colored stools, and the nurse recognizes the need for fecal occult blood testing (FOBT). What aspect of the client's current health status would contraindicate FOBT? A. Gastroesophageal reflux disease (GERD) B. Peptic ulcers C. Hemorrhoids D. Recurrent nausea and vomiting

Hemorrhoids

A client is scheduled for the creation of a continent ileostomy. What dietary guidelines should the nurse encourage during the weeks following surgery? A. A minimum of 30 g of soluble fiber daily B. Increased intake of free water and clear juices C. High intake of strained fruits and vegetables D. A high-calorie, high-residue diet

High intake of strained fruits and vegetables

A nurse is presenting an educational event to a local community group. When speaking about colorectal cancer, what risk factor should the nurse cite? A. High levels of alcohol consumption B. History of bowel obstruction C. History of diverticulitis D. Longstanding psychosocial stress

High levels of alcohol consumption

A nurse is taking a health history on a new patient who has been experiencing unexplained paresthesia. What question should guide the nurses assessment of the patients altered sensations?

How does the feeling in the affected extremity compare with the feeling in the unaffected extremity?

The orthopedic nurse should assess for signs and symptoms of Volkmann contracture if a client has fractured which of the following bones?

Humerus

A nurse has auscultated a client's abdomen and noted one or two bowel sounds in a 2-minute period of time. How should the nurse document the client's bowel sounds? A. Normal B. Hypoactive C. Hyperactive D. Paralytic ileus

Hypoactive

A critical care nurse is caring for a client diagnosed with acute pancreatitis. The nurse knows this client should be started on parenteral nutrition (PN) after what indications? A. 5% deficit in body weight compared to pre-illness weight and increased caloric need B. Calorie deficit and muscle wasting combined with low electrolyte levels C. Inability to take in adequate oral food or fluids within 7 days D. Significant risk of aspiration coupled with decreased level of consciousness

Inability to take in adequate oral food or fluids within 7 days

A nurse is caring for a client who is acutely ill and has included vigilant oral care in the client's plan of care. What factor increases this client's risk for dental caries? A. Hormonal changes brought on by the stress response cause an acidic oral environment B. Systemic infections frequently migrate to the teeth C. Hydration that is received intravenously lacks fluoride D. Inadequate nutrition and decreased saliva production can cause cavities

Inadequate nutrition and decreased saliva production can cause cavities

A client who has been experiencing changes in his bowel function is scheduled for a barium enema. What instruction should the nurse provide for postprocedure recovery? A. Remain NPO for 6 hours postprocedure. B. Administer a Fleet enema to cleanse the bowel of the barium. C. Increase fluid intake to evacuate the barium. D. Avoid dairy products for 24 hours' postprocedure.

Increase fluid intake to evacuate the barium.

An older adult woman's current medication regimen includes alendronate. What outcome would indicate successful therapy?

Increased bone mass

During a client's scheduled home visit, an older adult client has stated to the community health nurse that the client has been experiencing hemorrhoids of increasing severity in recent months. The nurse should recommend which of the following? A. Regular application of an OTC antibiotic ointment B. Increased fluid and fiber intake C. Daily use of OTC glycerin suppositories D. Use of an NSAID to reduce inflammation

Increased fluid and fiber intake

A clinic nurse is caring for a client with suspected gout. While describing the pathophysiology of gout to the client, what should the nurse explain?

Increased uric acid levels

The nursing care plan for a client in traction specifies regular assessments for venous thromboembolism (VTE). When assessing a client's lower limbs, what sign or symptom is suggestive of deep vein thrombosis (DVT)?

Increased warmth of the calf

A medical nurse who is caring for a client being discharged home after a radical neck dissection has collaborated with the home health nurse to develop a plan of care for this client. What is a priority psychosocial outcome for this client? A. Indicates acceptance of altered appearance and demonstrates positive self-image B. Freely expresses needs and concerns related to postoperative pain management C. Compensates effectively for alteration in ability to communicate related to dysarthria D. Demonstrates effective stress management techniques to promote muscle relaxation

Indicates acceptance of altered appearance and demonstrates positive self-image

A client with a documented history of allergies presents to the clinic. The client reports being frustrated by chronic nasal congestion, anosmia (inability to smell), and inability to concentrate. The nurse should identify which nursing diagnosis?

Ineffective individual coping with chronicity of condition

A nurse is planning the care of a client who has a long history of chronic pain, which has only recently been diagnosed as fibromyalgia. Which nursing diagnosis is most likely to apply to this client's care needs?

Ineffective role performance related to pain

A nurse is caring for a client who is receiving parenteral nutrition. When writing this client's plan of care, which of the following nursing diagnoses should be included? A. Risk for peripheral neurovascular dysfunction related to catheter placement B. Ineffective role performance related to parenteral nutrition C. Bowel incontinence related to parenteral nutrition D. Chronic pain related to catheter placement

Ineffective role performance related to parenteral nutrition

Which of the following is the most plausible nursing diagnosis for a client whose treatment for colon cancer has necessitated a colostomy? A. Risk for unstable blood glucose due to changes in digestion and absorption B. Unilateral neglect related to decreased physical mobility C. Risk for excess fluid volume related to dietary changes and changes in absorption D. Ineffective sexuality patterns related to changes in self-concept

Ineffective sexuality patterns related to changes in self-concept

A nurse is providing care for a client whose neck dissection surgery involved the use of a graft. When assessing the graft, the nurse should prioritize data related to what nursing diagnosis? A. Risk for disuse syndrome B. Unilateral neglect C. Risk for trauma D. Ineffective tissue perfusion

Ineffective tissue perfusion

A client is admitted to the medical unit with a diagnosis of intestinal obstruction. When planning this client's care, which of the following nursing diagnoses should the nurse prioritize? A. Ineffective tissue perfusion related to bowel ischemia B. Imbalanced nutrition: Less than body requirements related to impaired absorption C. Anxiety related to bowel obstruction and subsequent hospitalization D. Impaired skin integrity related to bowel obstruction

Ineffective tissue perfusion related to bowel ischemia

A nurse is assessing a client who has peptic ulcer disease. The client requests more information about the typical causes of Helicobacter pylori infection. What would it be appropriate for the nurse to instruct the client? A. Most affected clients acquired the infection during international travel. B. Infection typically occurs due to ingestion of contaminated food and water. C. Many people possess genetic factors causing a predisposition to H. pylori infection. D. The H. pylori microorganism is endemic in warm, moist climates.

Infection typically occurs due to ingestion of contaminated food and water.

A client presents to the clinic reporting vomiting and burning in the mid-epigastria. The nurse knows that in the process of confirming peptic ulcer disease, the health care provider is likely to order a diagnostic test to detect the presence of what? A. Infection with Helicobacter pylori B. Excessive stomach acid secretion C. An incompetent pyloric sphincter D. A metabolic acid-base imbalance

Infection with Helicobacter pylori

1) A nurse is caring for a client who is scheduled for a colonoscopy and whose preparation will include polyethylene glycol electrolyte lavage prior to the procedure. The presence of what health problem would contraindicate the use of this form of bowel preparation? A. Inflammatory bowel disease B. Intestinal polyps C. Diverticulitis D. Colon cancer

Inflamatory bowel disease

A client has experienced symptoms of dumping syndrome following gastric surgery. To what physiologic phenomenon does the nurse attribute this syndrome? A. Irritation of the phrenic nerve due to diaphragmatic pressure B. Chronic malabsorption of iron and vitamins A and C C. Reflux of bile into the distal esophagus D. Influx of extracellular fluid into the small intestine

Influx of extracellular fluid into the small intestine

The nurse is caring for a client who has a diagnosis of AIDS. Inspection of the client's mouth reveals the new presence of white lesions on the client's oral mucosa. What is the nurse's most appropriate response? A. Encourage the client to gargle with salt water twice daily. B. Attempt to remove the lesions with a tongue depressor. C. Make a referral to the unit's dietitian. D. Inform the primary provider of this finding.

Inform the primary provider of this finding.

A nurse is initiating parenteral nutrition (PN) to a postoperative client who has developed complications. The nurse should initiate therapy by performing which of the following actions? A. Starting with a rapid infusion rate to meet the client's nutritional needs as quickly as possible B. Initiating the infusion slowly and monitoring the client's fluid and glucose tolerance C. Changing the rate of administration every 2 hours based on serum electrolyte values D. Increasing the rate of infusion at mealtimes to mimic the circadian rhythm of the body

Initiating the infusion slowly and monitoring the client's fluid and glucose tolerance

A patient has been experiencing an unexplained decline in knee function and has consequently been scheduled for arthrography. The nurse should teach the patient about what process?

Injection of a contrast agent into the knee joint prior to ROM exercises

A nurse is preparing to place a client's prescribed nasogastric tube. What anticipatory guidance should the nurse provide to the client? A. Insertion is likely to cause some gagging. B. Insertion will cause some short-term pain. C. A narrow-gauge tube will be inserted before being replaced with a larger-gauge tube. D. Topical anesthetics will be used to reduce discomfort during insertion.

Insertion is likely to cause some gagging.

A 35-year-old client presents at the emergency department with symptoms of a small bowel obstruction. In collaboration with the primary care provider, what intervention should the nurse prioritize? A. Insertion of a nasogastric tube B. Insertion of a central venous catheter C. Administration of a mineral oil enema D. Administration of a glycerin suppository and an oral laxative

Insertion of a nasogastric tube

A client has been admitted to the hospital after diagnostic imaging revealed the presence of a gastric outlet obstruction (GOO). What is the nurse's priority intervention? A. Administration of antiemetics B. Insertion of an NG tube for decompression C. Infusion of hypotonic IV solution D. Administration of proton pump inhibitors as prescribed

Insertion of an NG tube for decompression

The nurse is preparing to perform a client's abdominal assessment. What examination sequence should the nurse follow? A. Inspection, auscultation, percussion, and palpation B. Inspection, palpation, auscultation, and percussion C. Inspection, percussion, palpation, and auscultation D. Inspection, palpation, percussion, and auscultation

Inspection, auscultation, percussion, and palpation

The nurses comprehensive assessment of an older adult involves the assessment of the patients gait. How should the nurse best perform this assessment?

Instruct the patient to walk away from the nurse for a short distance and then toward the nurse.

A nurse is caring for a client who is being treated in the hospital for a spontaneous vertebral fracture related to osteoporosis. The nurse should address the nursing diagnosis of Acute Pain Related to Fracture by implementing what intervention?

Intermittent application of heat to the client's back

A nurse is discussing conservative management of tendonitis with a client. What is the nurse's best recommendation?

Intermittent application of ice and heat

A client is undergoing diagnostic testing for a tumor of the small intestine. What are the most likely symptoms that prompted the client to first seek care? A. Hematemesis and persistent sensation of fullness B. Abdominal bloating and recurrent constipation C. Intermittent pain and bloody stool D. Unexplained bowel incontinence and fatty stools

Intermittent pain and bloody stool

An older adult client sought care for the treatment of a swollen, painful knee joint. Diagnostic imaging and culturing of synovial fluid resulted in a diagnosis of septic arthritis. The nurse should prioritize what aspect of care?

Intravenous administration of antibiotics

A nurse is preparing to administer a client's intravenous fat emulsion simultaneously with parenteral nutrition (PN). What principle should guide the nurse's action? A. Intravenous fat emulsions may be infused simultaneously with PN through a Y-connector close to the infusion site and should not be filtered. B. The nurse should prepare for placement of another intravenous line, as intravenous fat emulsions may not be infused simultaneously through the line used for PN. C. Intravenous fat emulsions may be infused simultaneously with PN through a Y-connector close to the infusion site after running the emulsion through a filter. D. The intravenous fat emulsions can be piggy-backed into any existing IV solution that is infusing.

Intravenous fat emulsions may be infused simultaneously with PN through a Y-connector close to the infusion site and should not be filtered

A nurse is providing client education for a client with peptic ulcer disease secondary to chronic nonsteroidal anti-inflammatory drug (NSAID) use. The client has recently been prescribed misoprostol. What would the nurse be most accurate in informing the client about the drug? A. It reduces the stomach's volume of hydrochloric acid B. It increases the speed of gastric emptying C. It protects the stomach's lining D. It increases lower esophageal sphincter pressure

It protects the stomachs lining

A nurse is performing the health history and physical assessment of a client who has a diagnosis of rheumatoid arthritis (RA). What assessment finding is most consistent with the clinical presentation of RA?

Joint stiffness lasting longer than 1 hour, especially in the morning

The nurse educator on an orthopedic trauma unit is reviewing the safe and effective use of traction with some recent nursing graduates. What principle should the educator promote?

Knots in the rope should not be resting against pulleys.

An older adult patient has come to the clinic for a regular check-up. The nurses initial inspection reveals an increased thoracic curvature of the patients spine. The nurse should document the presence of which of the following?

Kyphosis

A nurse is educating a client with gout about lifestyle modifications that can help control the signs and symptoms of the disease. What recommendation should the nurse make?

Limiting intake of alcohol

The results of a nurses musculoskeletal examination show an increase in the lumbar curvature of the spine. The nurse should recognize the presence of what health problem?

Lordosis

A client is undergoing diagnostic testing for osteomalacia. Which of the following laboratory results are most suggestive of this diagnosis?

Low serum calcium and low phosphorus level

The nurse is caring for a client scheduled for a colonoscopy. The nurse should assist the client into what position during this diagnostic test? A. In a knee-chest position (lithotomy position) B. Lying prone with legs drawn toward the chest C. Lying on the left side with legs drawn toward the chest D. In a prone position with two pillows elevating the buttocks

Lying on the left side with legs drawn toward the chest

A nurse is talking with a client who is scheduled to have a hemicolectomy with the creation of a colostomy. The client admits to being anxious, and has many questions concerning the surgery, the care of a stoma, and necessary lifestyle changes. What nursing action is most appropriate? A. Reassure the client that the procedure is relatively low risk and that clients are usually successful in adjusting to an ostomy. B. Provide the client with educational materials that match the client's learning style. C. Encourage the client to write down these concerns and questions to bring forward to the surgeon. D. Maintain an open dialogue with the client and facilitate a referral to the wound-ostomy-continence (WOC) nurse.

Maintain an open dialogue with the client and facilitate a referral to the wound-ostomy-continence (WOC) nurse.

A client has been diagnosed with a small bowel obstruction and has been admitted to the medical unit. The nurse's care should prioritize which of the following outcomes? A. Preventing infection B. Maintaining skin and tissue integrity C. Preventing nausea and vomiting D. Maintaining fluid and electrolyte balance

Maintaining fluid and electrolyte balance

A nurse is caring for a client who has suffered an unstable thoracolumbar fracture. What goal should the nurse prioritize during nursing care?

Maintaining spinal alignment

A nurse is providing care for a client who has osteomalacia. What major goal should guide the choice of medical and nursing interventions?

Maintenance of adequate levels of activated vitamin D

A client is recovering in the hospital following gastrectomy. The nurse notes that the client has become increasingly difficult to engage and has had several angry outbursts at staff members in recent days. The nurse's attempts at therapeutic dialogue have been rebuffed. What is the nurse's most appropriate action? A. Ask the client's primary provider to liaise between the nurse and the client. B. Delegate care of the client to a colleague. C. Limit contact with the client in order to provide privacy. D. Make appropriate referrals to services that provide psychosocial support.

Make appropriate referrals to services that provide psychosocial support.

A nurse is creating a teaching plan for a client who has a recent diagnosis of scleroderma. Which topics should the nurse address during health education?

Management of Raynaud-type symptoms Exercise Skin care

A client has a gastrostomy tube that has been placed to drain stomach contents by low intermittent suction. What is the nurse's priority during this aspect of the client's care? A. Measure and record drainage. B. Monitor drainage for change in color. C. Titrate the suction every hour. D. Feed the client via the G tube as prescribed.

Measure and record drainage.

A nurse is caring for clients in a stroke rehabilitation facility. Damage to what area of the brain will most affect a client's ability to swallow? A. Temporal lobe B. Medulla oblongata C. Cerebellum D. Pons

Medulla Oblongata

A client has been diagnosed with a malignancy of the oral cavity and is undergoing oncologic treatment. The oncologic nurse is aware that the prognosis for recovery from head and neck cancers is often poor because of what characteristic of these malignancies? A. Radiation therapy often results in secondary brain tumors. B. Surgical complications are exceedingly common. C. Diagnosis rarely occurs until the cancer is end stage. D. Metastases are common and respond poorly to treatment.

Metastases are common and respond poorly to treatment.

A nurse is providing care for a client who has just been diagnosed with early-stage rheumatoid arthritis (RA). The nurse should anticipate the administration of which medication?

Methotrexate

A client with GERD has undergone diagnostic testing and it has been determined that increasing the pace of gastric emptying may help alleviate symptoms. The nurse should anticipate that the client may be prescribed what drug? A. Metoclopramide B. Omeprazole C. Lansoprazole D. Calcium carbonate

Metoclopramide

A client with diabetes is attending a class on the prevention of associated diseases. What action should the nurse teach the client to reduce the risk of osteomyelitis?

Monitor and control blood glucose levels.

A client who underwent a gastric resection 3 weeks ago is having their diet progressed on a daily basis. Following the latest meal, the client reports dizziness and palpitations. Inspection reveals that the client is diaphoretic. What is the nurse's best action? A. Insert a nasogastric tube promptly. B. Reposition the client supine. C. Monitor the client closely for further signs of dumping syndrome. D. Assess the client for signs and symptoms of aspiration.

Monitor the client closely for further signs of dumping syndrome.

A client with systemic lupus erythematosus (SLE) asks the nurse why the client has to come to the office so often for "check-ups." Which rationale for frequent office visits would be best for the nurse to mention?

Monitoring the disease process and how well the prescribed treatment is working

A nurse is assessing a client who reports a throbbing, burning sensation in the right foot. The client states that the pain is worst during the day but notes that the pain is relieved with rest. The nurse should recognize the signs and symptoms of what health problem?

Morton neuroma

A nurse is assessing a client with rheumatoid arthritis. The client expresses the intent to pursue complementary and alternative medicine (CAM) therapies. Which fact should underlie the nurse's response to the client?

Most CAM therapies lack sufficient evidence to support them.

A nurse is caring for an older adult who has been diagnosed with geriatric failure to thrive. This patients prolonged immobility creates a risk for what complication?

Muscle atrophy

A client who had a total hip replacement two days ago reports new onset calf tenderness to the nurse. Which action should the nurse take?

Notify the health care provider.

A nurse is caring for a client who has a diagnosis of GI bleed. During shift assessment, the nurse finds the client to be tachycardic and hypotensive, and the client has an episode of hematemesis while the nurse is in the room. In addition to monitoring the client's vital signs and level of conscious, what would be a priority nursing action for this client? A. Place the client in a prone position. B. Provide the client with ice water to slow any GI bleeding. C. Prepare for the insertion of an NG tube. D. Notify the health care provider.

Notify the healthcare provider

A 10-year-old client is growing at a rate appropriate for the client's age. Which cells are responsible for the secretion of bone matrix, which eventually results in bone growth?

Osteoblasts

An 80-year-old man in a long-term care facility has a chronic leg ulcer and states that the area has become increasingly painful in recent days. The nurse notes that the site is now swollen and warm to the touch. The client should undergo diagnostic testing for what health problem?

Osteomyelitis

A client with diabetes has been diagnosed with osteomyelitis. The nurse observes that the client's right foot is pale and mottled, cool to touch, with a capillary refill of greater than 3 seconds. The nurse should suspect what type of osteomyelitis?

Osteomyelitis with vascular insufficiency

A client has been admitted to a medical unit with a diagnosis of polymyalgia rheumatica (PMR). The nurse should be aware of which aspects of PMR?

PMR has an association with the genetic marker HLA-DR4. Immunoglobulin deposits occur in PMR. PMR occurs predominately in Caucasians.

A bone biopsy has just been completed on a patient with suspected bone metastases. What assessment should the nurse prioritize in the immediate recovery period?

Pain

A client with a history of arthritis is being discharged to home after right wrist surgery, and the nurse reviews nonopioid pain relief measures. Which intervention(s) would best address the needs of this client?

Paraffin bath Nonsteroidal anti-inflammatory drugs (NSAIDs) Splint or brace

A nurse is promoting increased protein intake to enhance a client's wound healing. What is the enzyme that will initiate the digestion of the protein that the client consumes? A. Pepsin B. Intrinsic factor C. Lipase D. Amylase

Pepsin

An advanced practice nurse is assessing the size and density of a client's abdominal organs. If the results of palpation are unclear to the nurse, what assessment technique should be implemented? A. Percussion B. Auscultation C. Inspection D. Rectal examination

Percussion

Diagnostic imaging and physical assessment have revealed that a client with peptic ulcer disease has suffered a perforated ulcer. The nurse recognizes that emergency interventions must be performed as soon as possible in order to prevent the development of what complication? A. Peritonitis B. Gastritis C. Gastroesophageal reflux D. Acute pancreatitis

Peritonitis

A client is being assessed for a suspected deficit in intrinsic factor synthesis. What diagnostic or assessment finding is the most likely rationale for this examination of intrinsic factor production? A. Muscle wasting B. Chronic jaundice in the absence of liver disease C. The presence of fat in the client's stool D. Persistently low hemoglobin and hematocrit

Persistently low hemoglobin and hematocrit

What nursing intervention should the nurse prioritize to facilitate healing in a client who has suffered a hip fracture?

Place a pillow between the client's legs when turning.

A nurse is caring for a client who is postoperative day 1 following a total arthroplasty of the right hip. How should the nurse position the client?

Place a pillow between the legs.

A client has undergone surgery for oral cancer and has just been extubated in postanesthetic recovery. What nursing action best promotes comfort and facilitates spontaneous breathing for this client? A. Placing the client in a left lateral position B. Administering opioids as prescribed C. Placing the client in Fowler position D. Teaching the client to use the client-controlled analgesia (PCA) system

Placing the client in Fowler position

A client who underwent surgery for esophageal cancer is admitted to the critical care unit following postanesthetic recovery. What should the nurse include in the client's immediate postoperative plan of care? A. Teaching the client to self-suction B. Performing chest physiotherapy to promote oxygenation C. Positioning the client to prevent gastric reflux D. Providing a regular diet as tolerated

Positioning the client to prevent gastric reflux

A client is involved in a motorcycle accident and injures an arm. The health care provider diagnoses the man with an intra-articular fracture and splints the injury. The nurse implements the teaching plan developed for this client. What sequela of intra-articular fractures should the nurse describe regarding this client?

Posttraumatic arthritis

A nurse is caring for an older adult who has been experiencing severe Clostridium difficile-related diarrhea. When reviewing the client's most recent laboratory tests, the nurse should prioritize what finding? A. White blood cell level B. Creatinine level C. Hemoglobin level D. Potassium level

Potassium level

A client was fitted with an arm cast after fracturing the humerus. Twelve hours after the application of the cast, the client tells the nurse that the injured arm hurts. Analgesics do not relieve the pain. What would be the most appropriate nursing action?

Prepare the client for opening or bivalving of the cast.

Results of a client's preliminary assessment prompted an examination of the client's carcinoembryonic antigen (CEA) levels, which have come back positive. What is the nurse's most appropriate response to this finding? A. Perform a focused abdominal assessment. B. Prepare to meet the client's psychosocial needs. C. Liaise with the nurse practitioner to perform an anorectal examination. D. Encourage the client to adhere to recommended screening protocols.

Prepare to meet the client's psychosocial needs.

A nurse's plan of care for a client with rheumatoid arthritis includes several exercise-based interventions. What goal should the nurse prioritize?

Preserve or increase range of motion while limiting joint stress.

A nurse on the orthopedic unit is assessing a patients peroneal nerve. The nurse will perform this assessment by doing which of the following actions?

Pricking the skin between the great and second toe

A client who has had an amputation is being cared for by a multidisciplinary rehabilitation team. What is the primary goal of this multidisciplinary team?

Promote the client's highest possible level of function.

A client who has had a radical neck dissection is being prepared for discharge. The discharge plan includes referral to an outpatient rehabilitation center for physical therapy. What should the goals of physical therapy for this client include? A. Muscle training to relieve dysphagia B. Relieving nerve paralysis in the cervical plexus C. Promoting maximum shoulder function D. Alleviating achalasia by decreasing esophageal peristalsis

Promoting maximum shoulder function

A client has undergone rigid fixation for the correction of a mandibular fracture suffered in a fight. What area of care should the nurse prioritize when planning this client's discharge education? A. Resumption of activities of daily living B. Pain control C. Promotion of adequate nutrition D. Strategies for promoting communication

Promotion of adequate nutrition

A nurse is assessing the neurovascular status of a client who has had a leg cast recently applied. The nurse is unable to palpate the client's dorsalis pedis or posterior tibial pulse and the client's foot is pale. What is the nurse's most appropriate action?

Promptly inform the primary care provider.

A nurse is caring for a client who has undergone neck resection with a radial forearm free flap. The nurse's most recent assessment of the graft reveals that it has a bluish color and that mottling is visible. What is the nurse's most appropriate action? A. Document the findings as being consistent with a viable graft. B. Promptly report these indications of venous congestion. C. Closely monitor the client and reassess in 30 minutes. D. Reposition the client to promote peripheral circulation.

Promptly report these indications of venous congestion.

A nurse is caring for a client who has a leg cast. The nurse observes the client using a pencil to scratch the skin under the edge of the cast. How should the nurse respond to this observation?

Provide a fan to blow cool air into the cast to relieve itching

A client has just been diagnosed with acute gastritis after presenting in distress to the emergency department with abdominal symptoms. What would be the nursing care most needed by the client at this time? A. Teaching the client about necessary nutritional modification B. Helping the client weigh treatment options C. Teaching the client about the etiology of gastritis D. Providing the client with physical and emotional support

Providing the client with physical and emotional support

A client has just been diagnosed with acute gastritis after presenting in distress to the emergency department with abdominal symptoms. Which of the following actions should the nurse prioritize? A. Teaching the client about necessary nutritional modification B. Helping the client weigh treatment options C. Teaching the client about the etiology of gastritis D. Providing the client with physical and emotional support

Providing the client with physical and emotional support

The nurse is preparing to care for a client who has scleroderma. The nurse refers to resources that describe CREST syndrome. Which condition is a component of CREST syndrome?

Raynaud phenomenon

A client has been diagnosed with an esophageal diverticulum after undergoing diagnostic imaging. When taking the health history, the nurse should expect the client to describe what sign or symptom? A. Burning pain on swallowing B. Regurgitation of undigested food C. Symptoms mimicking a myocardial infarction D. Chronic parotid abscesses

Regurgitation of undigested food

A client's fracture is healing and compact bone is replacing spongy bone around the periphery of the fracture. This process characterizes what phase of the bone healing process?

Remodeling

A nurse caring for a client with a newly created ileostomy assesses the client and notes that the client has not had ostomy output for the past 12 hours. The client also reports worsening nausea. What is the nurse's priority action? A. Facilitate a referral to the wound-ostomy-continence (WOC) nurse. B. Report signs and symptoms of obstruction to the health care provider. C. Encourage the client to mobilize in order to enhance motility. D. Contact the health care provider and obtain a swab of the stoma for culture.

Report signs and symptoms of obstruction to the health care provider.

A nurse is caring for a client who is postoperative day 1 following neck dissection surgery. The nurse is performing an assessment of the client and notes the presence of high-pitched adventitious sounds over the client's trachea on auscultation. The client's oxygen saturation is 90% by pulse oximetry with a respiratory rate of 31 breaths per minute. What is the nurse's most appropriate action? A. Encourage the client to perform deep breathing and coughing exercises hourly. B. Reposition the client into a prone or semi-Fowler position and apply supplementary oxygen by nasal cannula. C. Activate the emergency response system. D. Report this finding promptly to the health care provider and remain with the client.

Report this finding promptly to the health care provider and remain with the client.

A client with rheumatoid arthritis comes to the clinic reporting pain in the joint of his right great toe and is eventually diagnosed with gout. When planning teaching for this client, what management technique should the nurse emphasize?

Restrict consumption of foods high in purines.

While performing an assessment, the nurse notes that a client has soft subcutaneous nodules along the extensor tendons of the fingers. Which disorder does this client most likely have?

Rheumatoid arthritis

A client's decreased mobility has been attributed to an autoimmune reaction originating in the synovial tissue, which caused the formation of pannus. This client has been diagnosed with which health problem?

Rheumatoid arthritis (RA)

A client has been scheduled for a urea breath test in one month's time. What nursing diagnosis most likely prompted this diagnostic test? A. Impaired dentition related to gingivitis B. Risk for impaired skin integrity related to peptic ulcers C. Imbalanced nutrition: Less than body requirements related to enzyme deficiency D. Diarrhea related to Clostridium difficile infection

Risk for impaired skin integrity related to peptic ulcers

A nurse is writing a care plan for a client admitted to the emergency department (ED) with an open fracture. The nurse will assign priority to what nursing diagnosis for a client with an open fracture of the radius?

Risk for infection

A 16-year-old presents at the emergency department reporting right lower quadrant pain and is subsequently diagnosed with appendicitis. When planning this client's nursing care, the nurse should prioritize what nursing diagnosis? A. Imbalanced nutrition: Less than body requirements related to decreased oral intake B. Risk for infection related to possible rupture of appendix C. Constipation related to decreased bowel motility and decreased fluid intake D. Chronic pain related to appendicitis

Risk for infection related to possible rupture of appendix

A nurse is caring for a client with a subclavian central line who is receiving parenteral nutrition (PN). In preparing a care plan for this client, what nursing diagnosis should the nurse prioritize? A. Risk for activity intolerance related to the presence of a subclavian catheter B. Risk for infection related to the presence of a subclavian catheter C. Risk for functional urinary incontinence related to the presence of a subclavian catheter D. Risk for sleep deprivation related to the presence of a subclavian catheter

Risk for infection related to the presence of a subclavian catheter

A client with rheumatic disease has developed a gastrointestinal (GI) bleed. The nurse caring for the client should further assess for medications that typically exacerbate this condition. Which medication applies?

Salicylate therapy

A client presents at a clinic reporting back pain that goes all the way down the back of the leg to the foot. The nurse should document the presence of what type of pain?

Sciatica

A 40-year-old woman was diagnosed with Raynaud phenomenon several years earlier and has sought care because of a progressive worsening of her symptoms. The client also states that many of her skin surfaces are "stiff, like the skin is being stretched from all directions." The nurse should recognize the need for medical referral for the assessment of what health problem?

Scleroderma

A nurses assessment of a teenage girl reveals that her shoulders are not level and that she has one prominent scapula that is accentuated by bending forward. The nurse should expect to read about what health problem in the patients electronic health record?

Scoliosis

A patient is receiving ongoing nursing care for the treatment of Parkinsons disease. When assessing this patients gait, what finding is most closely associated with this health problem?

Shuffling gait

The nurse is caring for a client with gastrointestinal symptoms who reports being under a significant amount of stress at home and at work. Which gastrointestinal effect of stress should the nurse anticipate is affecting this client? A. Increased gastric acid secretion B. Slowed peristalsis C. Increased enteric blood flow D. Relaxed sphincter muscles

Slowed peristalsis

A nursing educator is reviewing the risk factors for osteoporosis with a group of recent graduates. What of the following risk factors should the educator describe?

Small frame and female sex

A nurse is completing a health history on a client whose diagnosis is chronic gastritis. Which of the data should the nurse consider most significantly related to the etiology of the client's health problem? A. Consumes one or more protein drinks daily. B. Takes over-the-counter antacids frequently throughout the day. C. Smokes one pack of cigarettes daily. D. Reports a history of social drinking on a weekly basis.

Smokes one pack of cigarettes daily.

A nurse is assessing a child who has a diagnosis of muscular dystrophy. Assessment reveals that the childs muscles have greater-than-normal tone. The nurse should document the presence of which of the following?

Spasticity

A client presents to a clinic reporting a leg ulcer that isn't healing; subsequent diagnostic testing suggests osteomyelitis. The nurse is aware that the most common pathogen to cause osteomyelitis is:

Staphylococcus aureus.

A nurse is caring for an 83-year-old client who is being assessed for recurrent and intractable nausea. What age-related change to the GI system may be a contributor to the client's health issues? A. Stomach emptying takes place more slowly. B. The villi and epithelium of the small intestine become thinner. C. The esophageal sphincter becomes incompetent. D. Saliva production decreases.

Stomach emptying takes place more slowly.

A nurse is caring for a client hospitalized with an exacerbation of chronic gastritis. What health promotion topic should the nurse emphasize? A. Strategies for maintaining an alkaline gastric environment B. Safe technique for self-suctioning C. Techniques for positioning correctly to promote gastric healing D. Strategies for avoiding irritating foods and beverages

Strategies for avoiding irritating foods and beverages

A client has returned to the medical unit after a barium enema. When assessing the client's subsequent bowel patterns and stools, what finding would warrant reporting to the health care provider? A. Large, wide stools B. Milky white stools C. Three stools during an 8-hour period of time D. Streaks of blood present in the stool

Streaks of blood present in the stool

A nurse is collaborating with the physical therapist to plan the care of a client with osteomyelitis. What principle should guide the management of activity and mobility in this client?

Stress on the weakened bone must be avoided.

A 32-year-old client comes to the clinic reporting shoulder tenderness, pain, and limited movement. Upon assessment the nurse finds edema. An MRI shows hemorrhage of the rotator cuff tendons and the client is diagnosed with impingement syndrome. What action should the nurse recommend in order to promote healing?

Support the affected arm on pillows at night.

A nurse is caring for a client with a bone tumor. The nurse is providing education to help the client reduce the risk for pathologic fractures. What should the nurse teach the client?

Support the affected extremity with external supports such as splints.

A nurse is providing care for a client who has a rheumatic disorder. The nurse's focused assessment includes the client's mood, behavior, level of consciousness, and neurologic status. Which diagnosis is most likely for this client?

Systemic lupus erythematosus (SLE)

A client who experienced a large upper gastrointestinal (GI) bleed due to gastritis has had the bleeding controlled and is now stable. For the next several hours, the nurse caring for this client should assess for what signs and symptoms of recurrence? A. Tachycardia, hypotension, and tachypnea B. Tarry, foul-smelling stools C. Diaphoresis and sudden onset of abdominal pain D. Sudden thirst, unrelieved by oral fluid administration

Tachycardia, hypotension, and tachypnea

A nurse is caring for a client receiving skeletal traction. Due to the client's severe limits on mobility, the nurse has identified a risk for atelectasis or pneumonia. What intervention should the nurse provide in order to prevent these complications?

Teach the client to perform deep breathing and coughing exercises.

A client has just been diagnosed with a spondyloarthropathy. Which nursing intervention should the nurse prioritize?

Teaching about symptom management

The nurse is providing health education to a client with a gastrointestinal disorder. What should the nurse describe as a major function of the GI tract? A. The breakdown of food particles into cell form for digestion B. The maintenance of fluid and acid-base balance C. The absorption into the bloodstream of nutrient molecules produced by digestion D. The control of absorption and elimination of electrolytes

The absorption into the bloodstream of nutrient molecules produced by digestion

A nurse is caring for a client who has had a plaster arm cast applied. Immediately after application, the nurse should provide what teaching to the client?

The cast will only have full strength when dry.

A client's sigmoidoscopy has been successfully completed and the client is preparing to return home. What teaching point should the nurse include in the client's discharge education? A. The client should drink at least 2 liters of fluid in the next 12 hours. B. The client can resume a normal routine immediately. C. The client should expect fecal urgency for several hours. D. The client can expect some scant rectal bleeding.

The client can resume a normal routine immediately.

A client with a history of peptic ulcer disease has presented to the emergency department (ED) in distress. What assessment finding would lead the ED nurse to suspect that the client has a perforated ulcer? A. The client has abdominal bloating that developed rapidly. B. The client has a rigid, "board-like" abdomen that is tender. C. The client is experiencing intense lower right quadrant pain. D. The client is experiencing dizziness and confusion with no apparent hemodynamic changes.

The client has a rigid, "board-like" abdomen that is tender.

A client has recently received a diagnosis of gastric cancer; the nurse is aware of the importance of assessing the client's level of anxiety. Which of the following actions is most likely to accomplish this? A. The nurse gauges the client's response to hypothetical outcomes. B. The client is encouraged to express fears openly. C. The nurse provides detailed and accurate information about the disease. D. The nurse closely observes the client's body language.

The client is encouraged to express fears openly.

A medical client's CA 19-9 levels have become available and they are significantly elevated. How should the nurse best interpret this diagnostic finding? A. The client may have cancer, but other GI disease must be ruled out. B. The client most likely has early-stage colorectal cancer. C. The client has a genetic predisposition to gastric cancer. D. The client has cancer, but the site is unknown.

The client may have cancer, but other GI disease must be ruled out.

A client with gastroesophageal reflux disease (GERD) has a diagnosis of Barrett esophagus with minor cell changes. What principle should be integrated into the client's subsequent care? A. The client will be monitored closely to detect malignant changes. B. Liver enzymes must be checked regularly, as H2 receptor antagonists may cause hepatic damage. C. Small amounts of blood are likely to be present in the stools and are not cause for concern. D. Antacids may be discontinued when symptoms of heartburn subside.

The client will be monitored closely to detect malignant changes.

. A nurse is caring for a 78-year-old client with a history of osteoarthritis (OA). When planning the client's care, what goal should the nurse prioritize?

The client will express satisfaction with the ability to perform ADLs.

A nurse is assessing a client for risk factors known to contribute to osteoarthritis. What assessment finding should the nurse interpret as a risk factor?

The client's body mass index is 34 (obese).

A client's screening colonoscopy revealed the presence of numerous polyps in the large bowel. What principle should guide the subsequent treatment of this client's health problem? A. Adherence to a high-fiber diet will help the polyps resolve. B. The client should be assured that this is a normal, age-related physiologic change. C. The client's polyps constitute a risk factor for cancer. D. The presence of polyps is associated with an increased risk of bowel obstruction.

The client's polyps constitute a risk factor for cancer.

A nurse is assessing a client who is receiving traction. The nurse's assessment confirms that the client is able to perform plantar flexion. What conclusion can the nurse draw from this finding?

The client's tibial nerve is functional.

A client's neck dissection surgery resulted in damage to the client's superior laryngeal nerve. What area of assessment should the nurse consequently prioritize? A. The client's swallowing ability B. The client's ability to speak C. The client's management of secretions D. The client's airway patency

The clients ability to swallow

A nurse at an outpatient surgery center is caring for a client who had a hemorrhoidectomy. What discharge education topics should the nurse address with this client? A. The appropriate use of antibiotics to prevent postoperative infection B. The correct procedure for taking a sitz bath C. The need to eat a low-residue, low-fat diet for the next 2 weeks D. The correct technique for keeping the perianal region clean without the use of water

The correct procedure for taking a sitz bath

A client's rheumatoid arthritis (RA) has failed to respond appreciably to first-line treatments and the primary provider has added prednisone to the client's drug regimen. What principle will guide this aspect of the client's treatment?

The drug should be used for as short a time as possible.

A nurse is admitting a client diagnosed with late-stage gastric cancer. The client's family is distraught and angry that the client was not diagnosed earlier in the course of her disease. What factor most likely contributed to the client's late diagnosis? A. Gastric cancer does not cause signs or symptoms until metastasis has occurred. B. Adherence to screening recommendations for gastric cancer is exceptionally low. C. Early symptoms of gastric cancer are usually attributed to constipation. D. The early symptoms of gastric cancer are usually not alarming or highly unusual.

The early symptoms of gastric cancer are usually not alarming or highly unusual.

A nurse is planning discharge teaching for a 21-year-old client with a new diagnosis of ulcerative colitis. When planning family assessment, the nurse should recognize that which of the following factors will likely have the greatest impact on the client's coping after discharge? A. The family's ability to take care of the client's special diet needs B. The family's ability to monitor the client's changing health status C. The family's ability to provide emotional support D. The family's ability to manage the client's medication regimen

The family's ability to provide emotional support

An orthopedic nurse is caring for a client who is postoperative day 1 following foot surgery. What nursing intervention should be included in the client's subsequent care?

The foot should be elevated in order to prevent edema.

Diagnostic testing of a client with a history of dyspepsia and abdominal pain has resulted in a diagnosis of gastric cancer. The nurse's anticipatory guidance should include what information? A. The possibility of surgery, chemotherapy and radiotherapy B. The possibility of needing a short-term or long-term colostomy C. The benefits of weight loss and exercise as tolerated during recovery D. The good prognosis for clients who are treated for gastric cancer

The possibility of surgery, chemotherapy and radiotherapy

A patients fracture is healing and callus is being deposited in the bone matrix. This process characterizes what phase of the bone healing process?

The reparative phase

A client has been experiencing occasional episodes of constipation and has been unable to achieve consistent relief by increasing physical activity and improving the client's diet. When introducing the client to the use of laxatives, what teaching should the nurse emphasize? A. The effect of laxatives on electrolyte levels B. The underlying causes of constipation C. The risk of fecal incontinence D. The risk of becoming laxative-dependent

The risk of becoming laxative-dependent

A client with a recent history of intermittent bleeding is undergoing capsule endoscopy to determine the source of the bleeding. When explaining this diagnostic test to the client, what advantage should the nurse describe? A. The entire peritoneal cavity can be visualized. B. The test allows for painless biopsy collection. C. The capsule is endoscopically placed in the intestine. D. The test is noninvasive.

The test is noninvasive

A nurse is providing a class on osteoporosis at the local center for older adults. Which statement related to osteoporosis is most accurate?

The use of corticosteroids increases the risk of osteoporosis.

A nurse is explaining a patients decreasing bone density in terms of the balance between bone resorption and formation. What dietary nutrients and hormones play a role in the resorption and formation of adult bones? Select all that apply.

Thyroid hormone Growth hormone Estrogen

An older adult has a diagnosis of Alzheimer disease and has recently been experiencing fecal incontinence. However, the nurse has observed no recent change in the character of the client's stools. What is the nurse's most appropriate intervention? A. Keep a food diary to determine the foods that exacerbate the client's symptoms. B. Provide the client with a bland, low-residue diet. C. Toilet the client on a frequent, scheduled basis. D. Liaise with the primary provider to obtain an order for loperamide.

Toilet the client on a frequent, scheduled basis.

A client is brought to the emergency department by ambulance after stepping in a hole and falling. While assessing the client the nurse notes that the client's right leg is shorter than the left leg; the right hip is noticeably deformed and the client is in acute pain. Imaging does not reveal a fracture. What is the most plausible explanation for this client's signs and symptoms?

Traumatic hip dislocation

When assessing a patients peripheral nerve function, the nurse uses an instrument to prick the fat pad at the top of the patients small finger. This action will assess which of the following nerves?

Ulnar

A nurse is caring for a newly admitted client with a suspected GI bleed. The nurse assesses the client's stool after a bowel movement and notes it to be a tarry-black color. This finding is suggestive of bleeding from what location? A. Sigmoid colon B. Upper GI tract C. Large intestine D. Anus or rectum

Upper GI tract

A nurse is creating a care plan for a client receiving nasogastric tube feedings. Which intervention should the nurse include? A. Check the gastric residual volume every 4 hours. B. Hold the tube feeding if the gastric residual volume is greater than 200 mL. C. Position client flat in bed during feedings. D. Use client assessment findings to determine tolerance of feedings.

Use client assessment findings to determine tolerance of feedings.

A 91-year-old client is slated for orthopedic surgery and the nurse is integrating gerontologic considerations into the client's plan of care. What intervention is most justified in the care of this client?

Use of a pressure-relieving mattress

A nurse in an oral surgery practice is working with a client scheduled for removal of an abscessed tooth. When providing discharge education, the nurse should recommend what action? A. Rinse the mouth with alcohol before bedtime for the next 7 days. B. Use warm saline to rinse the mouth as needed. C. Brush around the area with a firm toothbrush to prevent infection. D. Use a toothpick to dislodge any debris that gets lodged in the socket.

Use warm saline to rinse the mouth as needed.

A nurse is planning client education for a client being discharged home with a diagnosis of rheumatoid arthritis. The client has been prescribed antimalarials for treatment, so the nurse knows to teach the client to self-monitor for what adverse effect?

Visual changes

A nasogastric tube is being inserted in a client with the COVID virus. Which action should the nurse take? A. Place the client in a prone position. B. Administer bolus feedings. C. Place a mask over the client's nose. D. Wear personal protective equipment.

Wear personal protective equipment.

A nurse is preparing to discharge a client after recovery from gastric surgery. What is an appropriate discharge outcome for this client? A. Bowel movements maintain a loose consistency. B. Three large meals per day are tolerated. C. Weight is maintained or gained. D. High calcium diet is consumed.

Weight is maintained or gained.

A patient has just had an arthroscopy performed to assess a knee injury. What nursing intervention should the nurse implement following this procedure?

Wrap the joint in a compression dressing.

A client with an exacerbation of systemic lupus erythematosus (SLE) has been hospitalized on a medical unit. The nurse observes that the client expresses anger and irritation when the call bell isn't answered immediately. Which response would be the most appropriate?

You seem like you're feeling angry. Is that something that we could talk about?"

A nurse is caring for a patient who has been scheduled for a bone scan. What should the nurse teach the patient about this diagnostic test?

Youll be encouraged to drink water after the administration of the radioisotope injection.

A school nurse is assessing a student who was kicked in the shin during a soccer game. The area of the injury has become swollen and discolored. The triage nurse should organize care for a:

contusion.

A client has come to the clinic for a routine annual physical. The nurse practitioner notes a palpable, painless projection of bone at the client's shoulder. The projection appears to be at the distal end of the humerus. The nurse should suspect the presence of:

osteochondroma

The nurse is admitting a client whose medication regimen includes regular injections of vitamin B12. The nurse should question the client about a history of: A. total gastrectomy. B. bariatric surgery. C. diverticulitis. D. gastroesophageal reflux disease (GERD).

total gastrectomy


Set pelajaran terkait

Essential Theological Concepts that undergird Christian Morality

View Set

4. Tissues: Concept and Classification

View Set

Secretions of the Intestine, Gall bladder and Pancreas. (FT: Physiology)

View Set

Sage 50 - Ch. 3 Review Questions

View Set

Frasi idiomatiche con il verbo "avere" (cap. 2-CIAO), Avere "to have"

View Set

Chapter 31: Inflation, Disinflation, and Deflation

View Set

Interview: Why Do Some Brains Enjoy Fear?

View Set